You are on page 1of 68

2017-18 100 &

op kers
Class 12 T
By E ran culty
-JE Fa r
IIT enior emie .
S fP r es
o titut
Ins

CHEMISTRY
FOR JEE MAIN & ADVANCED
SECOND
EDITION

Exhaustive Theory
(Now Revised)

Formula Sheet
9000+ Problems
based on latest JEE pattern

2500 + 1000 (New) Problems


of previous 35 years of
AIEEE (JEE Main) and IIT-JEE (JEE Adv)

5000+Illustrations and Solved Examples


Detailed Solutions
of all problems available

Plancess Concepts
Topic Covered Tips & Tricks, Facts, Notes, Misconceptions,
Key Take Aways, Problem Solving Tactics
Amines and Aromatic
Compounds Containing PlancEssential
Nitrogen Questions recommended for revision
25. AMINES AND AROMATIC
COMPOUNDS CONTAINING
NITROGEN

1. INTRODUCTION
(a) Alkyl derivatives of NH3 are called Amines.
(b) If a hydrogen atom of NH3 is replaced by an alkyl group, then it is called a primary amine and possesses −NH2
(amino) group.
(c) If two hydrogen atoms of NH3 are replaced then it is called a secondary amine and it posseses NH (Imino)
group.
(d) If all hydrogen atoms of NH3 are replaced then it is called a tertiary amine and has a nitrile N group.
(e) N is in sp3 hybridisation and has tetrahedral geometry.
(f) The bond angle increases from ammonia to 3º amines. NH3 (107º ) < RNH2 < R 2NH < R3N

2. STRUCTURE AND BONDING

2 3
sp hybridized sp hybridized

CH3 H = CH3 CH3 =


H 108 CH3
1.47 Å

Figure 25.1: Hybridisation in primary and tertiary amine

(a) Trigonal pyramidal geometry


(b) High negative potential on N
CH3NH2 (CH3)3N

Figure 25.2: Bond Structure of primary and tertiary amine


2 5 . 2 | Amines and Aromatic Compounds Containing Nitrogen

(c) Amine N can be a stereogenic centre.

N H H N
R R
R’ R’

Figure 25.3: Amines as a Pair of enantiomer

(d) Enantiomers rapidly interconvert.

R
R R
N : N : : N
H H
R’ R’ H R’

The two mirror images are interconverted.

Planar transition state


Figure 25.4: Interconversion of Enantiomers

(e) Enantiomers of quaternary ammonium salts:

R” R”
+ +
N N
R R’” R’” R
R’ R’

Figure 25.5: Isomerism shown by quaternary ammonium salt

3. PREPARATION OF AMINES

3.1 Nucleophilic Substitution


(a) Hofmann’s Ammonolysis Reactions:

Direct Nucleophilic Substitution

H
S N2 +
:

NH3
:

+
R X + NH3 R N H R NH2 + NH4
_
H X 1° amine
H
S N2 +
:

H R’NH2
:

+
R X + R’NH2 R N R N H + R’NH3
_
R’ X R’
H
S N2 +

:
NH3

:
+
R X + NH3 R N H R NH2 + NH4 Chem i str y | 25.3
_
H X 1° amine
H
S N2 +

:
H R’NH2

:
+
R X + R’NH2 R N R N H + R’NH3
_
R’ X R’
2° amine

H
S N2 +

:
R’ R’2NH

:
+
R X + R’2NH3 R N R N R’ + R’NH3
_
R’ X R’
3° amine

R’

_
S N2
R _ X + R’3N R_ N
:
+ _ R’ X

_
R’
Drawback:
(i) RX limited to methyl and 1º alkyl halides.
(ii) Complicated by polyalkylation.
(iii) Useful for preparing 1º amines and quaternary ammonium salts.

CH3
_

-
CH3 _ Br + CH3 _ N CH3 _ N
:

+
Br
_
_

CH3 CH3
Quaternary ammonium salt

(b) Gabriel Phthalamide Synthesis:



:

O O O
: :
_ SN2
H_N: _: N: R _ N:
_ R_X
: :

OH: :O :O O
:

Alkylated imide
Phthalimide _
pKa = 10 +X
_
OH H2O
_
CO2
• R = Me or 1º alkyl
R _ NH2 +
_

1° amine CO2
Dicarboxylate
by product

3.2 Reduction of Nitrogen Containing Functional Groups


(a) From Nitro Componds
reduce
R — NO2  → R — NH2
2 5 . 4 | Amines and Aromatic Compounds Containing Nitrogen

Reducing Agents:
(i) H2/Pd-C
(ii) (1) Fe, HCl (2) OH–
(iii) (1) Zn, or Sn, or SnCl2 + HCl (2) OH–

(b) From Nitriles


CH3CH2CH2 − C ≡ N →
1) LiAlH
4 CH3CH2CH2 − CH2NH2
2) H O 2

Nitriles from alkyl halides:

CH3CH2CH2 − Br NaCN
→
S 2
CH3CH2CH2 − C ≡ N
N

(c) From Amides


O
=

[1] LiAlH4
C [2] H2O
RCH2-NH2
R NH2
1° amide 1° amide
O
=

[1] LiAlH4
C [2] H2O
RCH2-N-R’
R NHR’
_

H
2° amide 2° amide
O
=

[1] LiAlH4
C RCH2-N-R’
[2] H2O
_

R NR’2
R’
3° amide 3° amide

3.3 Reductive Amination

Examples: O H NH2
NH3
NaBH3CN
Amphetamine
a powerful stimulant

CH3 CH3
CH3NH2
O NaBH2CN H NCH3
2° amine
Methamphetamine
Retrosynthetic Analysis: OH OH
NHCH3 O
+ H2NCH3
CH3 CH3
2° amine
Methamphetamine
Chem i str y | 25.5

3.4 Rearrangement Reactions


(a) Hofmann’s Bromamide Reaction: O 1. X2, base, H2O
(e.g. NaOH)
C 2. 
R-NH2
R NH2
1° amide 1° amine

X = Cl, Br

Mechanism:
Θ Θ : X _X : Θ
: :

: :
: :
: OH H2O

:
:

O :O: O :O:

:
:

:
:

: :
H X X:
R N R N R N R N

:
:

H H Θ H Θ
H2O
: :

: :
:X: : OH
Θ

: :
:X:

H H2O H H
O

:
:
Θ Θ
O O
:

:O :O :O
O
:
:
::

:
::

C
:
:

H
N N N :O H

:
N
H  H
:

R R
:

R H O
:O H R
:

H
:

Isocyanate
H H H intermediate
:

:O
:O Θ
:

O O:
:
::

N
:

R H N
:

R H
H
Θ CO2 (g) H
:

:O
:

:O O O:
:
::
:

N
N N  R H
R H R H
H H

(b) Curtius Rearrangement: O


=

C  HO
R-NH2
R-N=C=O -CO
2

R N3 -N2 2

Mechanism:

Step 1 Preparation of azides:

O + O O
Na
+ _ +
_
_ NaCl + +
R Cl N=N=N R N=N=N R _ N=N
_ Nl
2 5 . 6 | Amines and Aromatic Compounds Containing Nitrogen

Step 2 Decomposition: O
__ + 
R=N=C=O
R N=N
_ Nl -N2

Reaction with water to the unstable carbamic acid derivative which will undergo spontaneous decarboxylation:

O O
O
H H R H H2N-R
R-N  O R 2R-NH2
N OH N O -CO2
H H
Carbamic acid
Isocyanates are versatile starting materials:
O
O R
R’NH N OR’
C

R-N  O H
-N2 O
R N3
RNH2
R
N NHR
H
(c) Leuckart Thiophenol Reaction:

N2Cl KS OR S OR
+
S S
S OR SH
Base
S
S OR SR


The Leuckart Thiophenol Reaction allows the preparation of thiophenols and corresponding thioethers from
anilines or their corresponding diazonium salts. The first step of the reaction of an aryl diazonium salt with a
potassium alkyl xanthate to give an aryl xanthate, which affords an aryl mercaptan upon the basic hydrolysis
or an aryl thioether upon warming.

(d) Ritter Reaction:


O
H2SO4
+R N
N
H2 O R
H
O
H2SO4
OH + R N
H2 O R N
H

The acid-induced nucleophilic addition of a nitrile to a carbonium ion, followed by the hydrolysis to the
corresponding amide.
Chem i str y | 25.7

Mechanism of the Ritter Reaction: Any substrate capable of generating a stable carbonium ion is a suitable
starting material; primary alcohols do not react under these conditions, with the exception of benzylic alcohols.

H
_
H
+
+
O Ol
H H
-H2O

+
CH2 H
+

The carbonium ion adds to the nitrile nitrogen to give a nitrilium ion intermediate, which undergoes hydrolysis to
the corresponding amide upon the aqueous work-up.

+
+ + N R N R N
_+ R

H H H
_ H _ H
N O
+
-H /+H
+
+N Ol +N O H
+

N
_ O
+ +
H H
R R R R

(e) Lossen’s Rearrangement:


O
2
O R Θ
1 OH 1 H2 O 1
R N R -N=C=O R -NH2 + CO2
H O

Mechanism:
O O HO-H
2 2
O R Θ O R 1 1
1 1
R N R N R CO2 + R -N=C=O
Θ
O OH O
H :OH2
isocyanate
intermediate
H decarboxylation
N O 1
R -NH2 + CO2
1
R H
O
:B

(f) Claisen Schmidt rearrangement:


O
=

RCOOH
HN3
R _ C _ N3 (i) 
RNH2 + CO2
H2SO4 (ii) H2O

Illustration 1: Prepare n-butylamine by Gabriel synthesis. (JEE MAIN)

Sol: This method is mainly used in preparation of Primary amine by use of alkyl halide and Potassium phthalimide.
After the attack of alkyl halide, Potassium phthalimide no longer behaves as a nucleophile. Product is cleaved by
reaction with base or hydrazine, which leads to a stable cyclic product. The by-product of this reaction is an acid.
2 5 . 8 | Amines and Aromatic Compounds Containing Nitrogen

O O

Θ  n-Bu-Br
N_ K N _ Bu-n
-KBr

O O
Θ 
1. aq. OH 2. H3O

COOH
n-BuNH2 +
COOH

Illustration 2: RCH2CH=O + [Intermediate]


H2/Ni
(B) (JEE MAIN)
or
H (A) NaCNBH3
_

N
Piperidine

Sol: Reaction between aldehyde and piperidine proceed via formation of carbinolamine which gets converted to
Iminium ion intermediate. On reduction with Raney Ni, tertiary amine is formed.

RCH2 _ CH=O + H _ N
:

OH
_

RCH2 _ CH2_ N
A carbinolamine

Reduction 
RCH2CH2 _N RCH2 _ CH=N

(B) Iminium ion


Intermediate

(A)

Illustration 3: Complete the following reactions: (JEE ADVANCED)

(A) (A)
+ NH3 (B) (C) (D)
O
Oxirane
(A)


Sol: H3N
:

SN 2 H3N NH3
O Θ H2N OH
O (B)
:

(A)
(Aminoethanol)
(B) is basic and
reacts further
O with (A)

O (HOCH2CH2)2NH or NH
:

OH OH
(HOCH2CH2)3N (C)
(D)
Chem i str y | 25.9

Illustration 4: Complete the following: (JEE ADVANCED)


NO2
NH4SH
(B)

NO2
(A)

Sol: It is an example of region selective reaction. One of the nitro group preferentially gets reduced by NH4 SH
(NH4 )2 S or NH4 SH reduces only one nitro group. (B) is

NH2
B is

NO2

4. PHYSICAL PROPERTIES OF AMINES


(a) Unlike most other organic compounds, amines are far more soluble in water, this is because all amines form
a stronger H–bond with water.
(b) Like ammonia, amines are polar compounds and apart from 3º amines, they can form intermolecular H–
bonds that’s why they have higher b.p.s’.
(c) The boiling points of amines are lesser than of alcohols and acids of comparable mol. weight because H–
bonding in amines is less pronounced in 1º and 2º than that in alcohols and carboxylic acids. This is because
nitrogen is less electronegative than oxygen.
(d) Boiling point of 1º, 2º and 3º amines follow the order.
1º > 2º > 3º amine
(e) Solubility in water follow the order.
1º > 2º > 3º amine
This is all due to H–Bonding.

Basicity of Amines: Due to basic nature, they react with acids to form salts.


R _ NH2 + H _ X R _NH3X Θ (Salt)
:


Ph _ NH2 + HCl R _NH3Cl Θ (Anilinium chloride)
:

Amine salts on treatment with bases such as NaOH regenerate the parent amine.
+ − •• −
RNH3 X − + OH → R NH2 + H2O + X

Relative Basicity of Amines: Compare basicities by using conjugate acid pK a values.


(a) An Amine and Ammonia
H − NH3+ pK a =
9.3 NH3 (weaker base)
CH3CH2 − NH3+ pK a =
10.8 CH3CH2NH2 ( strong base)
2 5 . 1 0 | Amines and Aromatic Compounds Containing Nitrogen

(i) Alkyl amines are stronger bases than NH3


(ii) Electron-donating alkyl group increase the electron density on N.

(b) An Alkylamine and an Arylamine


+
NH3
+
CH3CH3NH3

pKa = 4.6 pKa = 10.8


Aniline is a weaker base that ethylamine. Why?
_ + +
:

NH2 NH2 NH2


:

etc.

_:
••
CH3CH2 NH2 localized lone pair

(i) Electron donors increase basicity.



:

NH2
D D can be -NH2, -OH, -OR, -NHCOR, -R

(ii) Electron withdrawers decrease basicity.



:

NH2
W can be -X, -CHO, -COR, -COOR,
W
-COOH, -CN, -SO2H, -NO2, -NR3

p-nitroaniline Aniline p-toluidine

Increasing Basicity

Figure 25.6: Order of Relative basicity of Amine

Order of Relative basicity of Amine

(c) An Alkylamine and an Amide


_

:
:

:O :O:
Lone pair on N is
C C + resonance delocalized.
:

R NH2 R NH2
Chem i str y | 25.11

(i) Amides are less basic than alkyl amines.


(ii) Preferred site of protonation is on oxygen.
+

:
:O: : OH : OH : OH
C H A C C C +
+ :A-
+

:
:
R NH2 R NH2 R NH2 R NH2

:O: :O:
C H A C +
+ :A-
:

R NH2 R NH3

(d) Heterocyclic Aromatic Amines



+ + :
N N N N
:

H H
H H
pKa = 5.3 pyridine pKa = 0 pyrrole

(i) Pyridine – stronger base – lone pair localized


(ii) Pyrrole – weaker base – lone pair delocalized

(e) Effect of Hybridization


N N N N
H H
H H
pKa = 5.3 pyridine pKa = 11.1 piperidine

(i) Pyridine – weaker base – lone pair in sp2 orbital


(ii) Piperidine – stronger base – lone pair in sp3 orbital

Illustration 5: Give the decreasing order of boiling points for the following:  (JEE MAIN)
Me
I. Et2NH II. Me NH2 III. Et N
Me
Sol: Extent of hydrogen bonding decreases in the order (1º > 2º > 3º amine) thus boiling point also decreases in
the order (1º > 2º > 3º amine)
(II) > (I) > (III) (1º > 2º > 3º amine)

Illustration 6: Arrange the following in the decreasing order of their basic strength:  (JEE MAIN)
a. i. PhNH2 ii. EtNH2 iii. Et2NH iv. NH3
b. i. EtNH2 ii. PhNH2 iii. NH3 iv. PhCH2NH2 v. Et2N

Sol: Extent of hydrogen bonding decreases in the order (1º > 2º > 3º amine) thus boiling point also decreases in
the order (1º > 2º > 3º amine)
2 5 . 1 2 | Amines and Aromatic Compounds Containing Nitrogen

a. Aliphatic 2º amine > Aliphatic 1º amine > NH3: Aromatic amine


(iii) > (ii) > (iv) > (i)
b. (v) > (i) > (iv) > (iii) > (ii)

Illustration 7: Complete the following acid-base reaction and name the products. (JEE ADVANCED)

NH2
(i) Me + HCl (ii) Et3N+ HCl

Sol: (i)

NH3Cl Θ
(i) (Propanammonium chloride)
Me

(ii) Et3N+ Cl− (Triethylammonium chloride)

PLANCESS CONCEPTS

Preparation of amines:
•• reduction of nitro compounds to amines by metals(Fe, Sn, Zn) in dil HCl or SnCl2 or by hydrogen in
presence of Ni, Pt, Pd.
Misconception: LiAlH4 doesn’t reduce the nitro compounds to amines. Instead it reduces it to
R-N=N-R.
•• Amines can also be prepared by the Hoffmann ammonolysis reaction which is a SN2 reaction. NH3
reacts with the alkyl halide to give primary halides which in turn react with more alkyl halides to give
2º and 3 amines and then gives 4º ammonium salts.
Exception: Hoffmann ammonolysis reaction cannot be used to prepare aromatic amines.
•• Amines can also be produced by the reduction of nitriles with LiAlH4 or catalytic hydrogenation. But
note that only primary amines can be produced.
Misconception: Besides NaBH4 does not give primary amines.
•• Amines can also be prepared by the reduction of amides, oximes and azides by LiAlH4 . But again
only primary amines can be produced by this method.
•• Primary amines can also be produced by Gabriel phthalamide synthesis. In this pthalic anhydride
is converted to phthalamide which is then treated with an alkyl halide to give N-alkyl pthalamide.
Finally it is treated with hydrazine to give a primary amine.
Exception: primary aromatic amines cannot be prepared by Gabriel phthalamide synthesis because
aromatic halide doesn’t give a substitution reaction with phthalamide.
•• Primary amines can be prepared by Hoffmann bromamide reaction where the amide is treated with
Br2 in the presence of an alkali to give a primary amine. The intermediate species is nitrene which
undergoes intra rearrangement to give RNH2.
•• Some other methods to prepare primary amines are Lossen rearrangement, Curtius rearrangement,
Claisen-Schmidt rearrangement. All have one thing in common, i.e., they proceed through the
nitrene mechanism.
•• Amines can also be produced by the reductive amination of carbonyl compounds. Carbonyl
compounds give imines which on reduction with H2 in presence of Ni gives an amine. Aldehydes
give primary amines and ketones give secondary amines.
Vaibhav Krishnan (JEE 2009 AIR 22)
Chem i str y | 25.13

5. VARIOUS TESTS FOR AMINES


(a) Hofmann Mustard Oil Reaction (Test for 1º Amines)

S
||
Warm HgCl
2 → 2HCl + HgS + R −=
RNH2 +=
S =
C S  → RNH − C− SH  N =
C S
1º amine Dithioalkyl Alkyl
carbamic acid isothiocyanate

Alkyl isothiocyanates gives a smell like that a mustard oil and this is used as a test for 1º amines.

(b) Carbylamine Reaction (Test for 1º Amine): It consists of heating a mixture of 1º aliphatic or aromatic
amines and chloroform with alcoholic KOH solution. The reaction proceeds via carbene mechanism.
R – NH2 + CHCl3 + 3KOH (Alcoholic)
∆ –
R – N+ ≡ C + 3KCl + 3H2O

Mechanism:
Cl
Θ Θ Θ
OH H C Cl H2O + CCl3 :CCl2 + Cl
Cl

R – NH2 + : CCl2 RN+ ≡ C + 2HCl


Isocyanides have a very offensive smell. So, this reaction is use to distinguish 1º amines.

(c) Hinsberg’s Test (for distinguishing 1º, 2º and 3º amines): Three classes of amines are distinguished by
Hinsberg reagent test. The Hinsberg reagent is a benzene sulphonyl chloride (C6H5SO2Cl).

(d) Reaction of Hinsberg Reagent with Primary Amines: A primary amine forms a precipitate of N-alkyl
benzene sulphonamide with Hinsberg reagent. This precipitate is soluble in alkali.


RNH2 (primary amine) + C6H5SO2Cl (Hinsberg reagent) → R − NH − SO2 − C6H5 
NaOH
→ R − N−Na+ − SO2C6H5 (soluble)

(e) Reaction of Hinsberg Reagent with a Secondary Amine: Secondary amine reacts with the Hinsberg reagent
to form a precipitate N,N-dialkyl benzene sulphonamide. But this precipitate is insoluble in alkalis.
NaOH
R 2NH (sec ondary amine) + C6H5SO2Cl (Hinsberg reagent) → R 2NSO2C6H5 
→ Insoluble (no reaction)
(Precipitate)

(f) Reaction of Hinsberg Reagent with a Tertiary Amine: Tertiary amines do not react with Hinsberg reagent.

R 3N (Tertiary amine) + C6H5SO2Cl (Hinsberg reagent) → No reaction

(g) Reaction with Nitrous acid (For Distinguishing 1º, 2º and 3º amines): Nitrous acid (HNO2 or HONO)
reacts with aliphatic amines in a fashion that provides a useful test for distinguishing, primary, secondary and
tertiary amines.

1º-Amines + HONO (cold acidic solution) → Nitrogen Gas Evolution from a Clear Solution
2º-Amines + HONO (cold acidic solution) → An Insoluble Oil (N-Nitrosamine)
3º-Amines + HONO (cold acidic solution) → A Clear Solution (Ammonium Salt Formation)

Nitrous acid is a Bronsted acid of moderate strength (pKa = 3.3). Because it is insoluble, it is prepared immediately
before use in the following manner.
H2O, 0º
NaNO2 + H2SO4 H—O—N=O + NaHSO4
2 5 . 1 4 | Amines and Aromatic Compounds Containing Nitrogen

Under the acidic conditions of this reaction, all amines undergo reversible salt formation:
1 1
R R
2 2
 Θ
R N: + HX R N H X [X = HSO4 or NO2]
3 3
R R
This happens with 3º-amines, and the salts are usually in water. The reactions of nitrous acid with 1º- and 2º-
aliphatic amines may be explained by considering their behaviour with the nitrosonium cation, NO(+), an electrophilic
species present in acidic nitrous acid solution.

Mechanism of reaction of Nitrous acid with primary amine

H R

HNO2, 0  -H Tautomerism
R_NH Θ
:

:
:
:
2 R N N=O X :N N=O R-N=N-OH
N=O
H H

H
Alcohols  -H2O 
H2 O -N2
R-NN: XΘ

:
:
and R R-N=N-OH2
Alkenes

6. CHEMICAL REACTIONS OF AMINES


6.1 Reaction with Carbonyl Compounds
(a) Imine Formation


:

H  OH H
:

O OH Me H
:

H2N N



OH2
H
 H H
:

N N N
imine
Imines are formed when any primary amine reacts with an aldehyde or ketone under appropriate conditions.
Imine formation requires an acid catalyst, otherwise the reaction is very slow. The acid is needed for the
elimination of water.

(b) Enamine Formation:


H

:

 OH
:

O OH H
H
:

N N


OH2
 H
H
N N
:

N
enamine

Under the appropriate conditions, secondary amines react with carbonyl compounds to form enamines. Like
imine formation, the formation of an enamine also requires an acid catalyst for the removal of water.
Chem i str y | 25.15

6.2 Elimination
(a) Hofmann Elimination: When a primary amine bearing one of more beta hydrogens is treated with methyl
iodide, followed by aqueous silver oxide, followed by heat, the primary amine is converted to an alkene. This
reaction is known as Hofmann elimination, not to be confused with Hofmann rearrangement.
1. 3 eq. CH3I
2. aq. Ag2O
E.g.
CH3CH2NH2  3. ∆
→ CH2 = CH2

The net reaction is 1,2-elimination, hence the name Hofmann Elimination.

H2C CH2 NH2

H
+
H -NH
2

Hofmann elimination occurs in three stages.


+
3 CH3I
CH3CH2NH2 Stage 1 CH3CH2N(CH3)3I-

Stage 2 aq. Ag2O

 +
CH2 = CH2 CH3CH2N(CH3)3 -OH
Stage 3

Mechanism:
Stage 1:
CH3

NH2 + I -
:

CH3CH2 NH2 + CH3 I CH3CH2


+
CH3 CH3
+ +
:

CH3CH2 N H + H2NCH2CH3 CH3CH2 N: + H3NCH2CH3

H H
CH3 CH3
+
CH3CH2 N: + CH3 I CH3CH2 N H + I-

H CH3
CH3 CH3
+ +
:

CH3CH2 N H + H2NCH2CH3 CH3CH2 N: + H3NCH2CH3

CH3 CH3
CH3 CH3
+
CH3CH2 N: + CH3 I CH3CH2 N CH3 I -

CH3 CH3

Stage 2:
Ag2O (s) + H2O AgOH (s)

+
+ CH3CH2N(CH3)3I -
CH3CH2N(CH3)3 -OH + AgI (s) Ag+ + -OH
2 5 . 1 6 | Amines and Aromatic Compounds Containing Nitrogen

Stage 3:
+ +

:
H2C CH2 N(CH3)3 + - : OH

: :

: :
CH2 CH2 N(CH3)3 + H2O

H
CH2 = CH2 + N(CH3)3
Stage 3 is a 1,2-elimination via E1CB mechanism. Hofmann elimination is regioselective. Since the 1,2-elimination
in Stage 3 occurs via E1CB mechanism. Hofmann rule is used to predict the major product.
eg.
1. 3eq. CH3I
2. aq. Ag2O major
3. 
NH2
minor

(b) Cope’s Elimination: Tertiary amine oxides undergo the elimination of a dialkylhydroxylamine when they are
heated. This reaction is called the Cope elimination:
:

:O:-
:

:OH:
RCH CH N+_ CH
2 2 3 RCH=CH2 + :N CH3
150
CH3 An alkene CH3
A tertiary amine N,N-Dimethyl-
oxide hydroxylamine

The Cope elimination is a syn elimination and proceeds through a cyclic transition state:
R
CH-CH2 CH R
3

N R-CH=CH2 + N
H
:

+
: :

-:O CH H-O CH3


: :

Tertiary amine oxides are easily prepared by treating amines with hydrogen peroxide.
The Cope elimination is useful synthetically. Consider the following synthesis of methylenecyclohexane:
H : O:-
:

160C
:

CH2+(CH3)2NOH
N+
CH3
CH3

6.3 Reaction with Acid Halides and Anhydrides


O O
+
C + NH3 C + NH4 Z-
(2 equiv)
R Z R NH2
1 amide
O O
+
C + R’NH2 C + R’NH3 Z-
(2 equiv)
R Z 1 amide R NHR’
2 amide
O O
+
C + R’2NH C + R’2NH2 Z-
(2 equiv)
R Z 2 amide R NR’2
Z=Cl or OCOR 3 amide
Chem i str y | 25.17

6.4 Electrophilic Substitution Reaction for Aniline


Electrophilic substitution: Due to +M effect of –NH2 generate electron density at ortho and para position hence,
aniline active toward electrophilic substitution
(a) Bromination: Aniline reacts with bromine water at room temperature to give a white precipitate of
2,4,6-tribromoaniline.
NH2 NH2
Br Br
Br2/H2O
+ HBr2 + 3HBr

Aniline Br
If we have to prepare a monosubstituted aniline derivative. This can be done by protecting the –NH2 group by
acetylation with an acetic anhydride, then carrying out the desired substitution followed by the hydrolysis of
the substituted amide to the substituted amine.
O O
NH2 H-N-C-CH3 H-N-C-CH3 NH2
1
(CH3CO)2O Br2 2
Pyridine CH3COOH OH- or H+ 3
Aniline N-Phenylethanamide Br Br
(Major) 4-Bromoaniline
The lone pair of electrons on nitrogen of acetanilide interacts with oxygen atom due to resonance as shown
below:
:O: :O:
+
:

N C CH3 N=C CH3

Hence, the lone pair of electrons on nitrogen is less available for donation to the benzene ring by resonance.
Therefore, activating the effect of –NHCOCH3 group is less than that of amino group.
(b) Nitration: Direct nitration of aniline is not possible because in the strongly acidic medium, aniline is protonated
to form the anilinium ion which is meta directing.

NH2 NH2 NH2 NH2


NO2
HNO3 +H2SO4,
+ +
NO2
NO2 47% 2%
51%

However, by protecting the –NH2 group by acetylation reaction with acetic anhydride, the nitration reaction
can be controlled and the p-nitro derivative can be obtained as the major product.

NH2 NHCOCH3 NHCOCH3 NH2


-
OH or H+
(CH3CO)2O HNO3+H2SO4, 288K
Pyridine

NO2 NO2
p-Nitroacetanilide p-Nitroaniline
2 5 . 1 8 | Amines and Aromatic Compounds Containing Nitrogen

(c) Sulphonation: + - +

:
NH2 NH3 HSO4 NH2 NH3

H2SO4 453-473K

Anillinium hydrogensluphate -
SOH SO3
Sulphanilic acid Zwitter ion

PLANCESS CONCEPTS

Chemical reaction of amines:


•• Tips and tricks: Only primary amines react with carbonyl compounds to give imines while secondary
amines react with carbonyl compounds to give enamines.
•• Tips and tricks: In Hoffman elimination, the alkene which is substituted least is the major product while
in Cope’s elimination the alkene which is most substituted is the major product..
•• Tips and tricks: In the Hoffmann elimination, there is anti-elimination while in Cope’s elimination, there
occurs syn elimination.
•• In electrophilic substitution of bromine on aniline, if the medium is CS2 then only 1 Br is substituted while
in aqueous solution, the substitution of bromine occurs 3 times.
•• If bromination is to be done once in aqueous solution then acetylation of bromine is carried out due to
which the ring becomes less activated.
•• In nitration, the major products are para and metanitroaniline while the ortho product is very less. This is
because of the acid base reaction between aniline and nitrous acid due to which some of aniline becomes
meta directing.
Vaibhav Krishnan (JEE 2009 AIR 22)

Illustration 8: Complete the following reactions:  (JEE MAIN)

Me
(a) NH2
NaNO2 + HCl NH2
Me (B) Me HNO2
(b) (D)
Me
(A)
(C)

Sol: Attack of nitrous acid forms a primary carbocation which gets rearranged to more stable secondary carbocation.
Attack of nucleophile (OH-) result in formation of secondary alcohol which on treatment with acid forms alkene.
(a) NH2  
HNO2
Me Me CH2 Me Me
 More stable
(A) 1C
2C 

OH

OH

H Me Me
Me
Since the conditions
(B) are acidic, H2O is
Propene
lost (-H2O).
Chem i str y | 25.19

(b) Me 
Me CH2 1,2-Me
HNO2
Me NH2
Me shift
Me Me
(C) 1C

Me 1 2 3 4  Me
H Me OH
Me 
Me -H O Me Me
Me 2

2-Methyl
Me More stable
3C
OH
but-2-ene
(D)

Illustration 9: Explain:  (JEE ADVANCED)


Br

I. Dehydrohalogenation of
Me Me

Me 

II. Hofmann elimination of


Me NMe3
OH

 −
Sol: E2 elimination of an alkyl halide with base  Et O  gives mainly Saytzeff alkene (i.e., more-substituted alkene)
 
 
Alc.
I. Me Me + Me
KOH
(70%) (Mostly trans) + (30%)

II.  → 3% (cis and trans) + (97%)
 −
E2 elimination of an alkyl halide with the base  Et O  gives mainly a Saytzeff alkene (i.e., more-substituted alkene),
 
 
whereas 4º ammonium salt undergoes Hofmann elimination to give a less-substituted alkene, resulting from a loss
of a more acidic β -H (1º>2º>3º) called Hofmann’s rule.
Thus, the acidity of β –H is more important than the stability of the alkene that is formed.

Illustration 10: Give the products of the following by the application of Hofmann’s exhaustive methylation and
elimination:  (JEE ADVANCED)

Me

(i) (ii) (iii) HN O


N
N
H

Sol: This process involves thermal decomposition of quaternary ammonium hydroxide compounds to yield an
olefin and water is known as Hofmann’s exhaustive methylation reaction. This method has been employed in
identifying site of unsaturation present in the given compound.
2 5 . 2 0 | Amines and Aromatic Compounds Containing Nitrogen

(i) 
Me H  Me Me
MeI H
OH

AgOH 
N N N Me
Me Me
H Me

Me  Me
MeI
 OH
  AqOH
Me N Me
Me

(ii) 6 4
  5
 7 3

  8 2
N 9
  1
(Nona-1,4,8-triene)

O
(iii)   O

 
N
(Divinyl ether)
N

7. DIAZONIUM SALTS
7.1 Introduction
The diazonium salts have the general formula RN2X where R stands for an aryl group and X ion may be Cl, Br, HSO4,
BF3 etc.
Resonance of benzene diazonium ion is
   
:
:

:
:

NN : NN : NN : NN :

7.2 Preparation
273-278 K
PhNH + NaNO2 + HCl


Ph-NN Cl +NaCl+2H2O
Benzene diazonium
chloride (1)
Chem i str y | 25.21

7.3 Chemical Reactions Involving Displacement of Nitrogen


HOH ArOH+H
 Slow  CN
ArN2-X Ar-N2 Ar Ar-CN
-N2
+ X
ArX
X
Boiling
(a) Replacement by ‒OH group: ArN2Cl + HOH  → ArOH + N2 + HCl

(b) Replacement by H: ON reduction with sodium stannite (Na2SnO2) (alkaline SnCl2) or hypophosphorous acid
(H3PO2) or on warming with C2H5OH, ArN2Cl gives benzene.
ArN2Cl+H3PO2+H2O → ArH+N2+2CuCl2
ArN2Cl+Na2SnO2+H2O → ArH+N2+ Na2SnO3+HCl

(c) Sandmeyer reaction (replacement by halogen):


ArN2Cl+Cu2Cl2+HCl → ArCl+N2+2CuCl2
ArN2Br+Cu2Br2+HBr → ArBr+N2+2CuBr2

(d) Gattermann reaction:


Finely divided Cu/HCl
ArN2Cl → ArCl+N2
Finely divided Cu/HBr
ArN2Br → ArBr+N2
Boiling
(e) Iodo compounds: ArN2Cl+KI(Aq.)  → Ar–I +N2+KCl

(f) Balz-Schiemann reaction:



NaNO2+HBF4
Ar-N2BF4 
(Fluoroboric acid)
Ar-NH2 Tetrafluoroborate
273-298K
salt
ArF+BF3+N2

(g) Cyano compounds:


CuCN
ArN2Cl+KCN 
or Cu powder
→ ArCN+N2+KCl
ArN2Cl+KCN 
K [Cu(CN) ]
→ ArCN+N2+KCl
3 4

(h) Nitro compounds:


Cu O
ArN2Cl+HONO →
2 ArNO2+N2+HCl
ArN2Cl+CuNO2 

→ ArNO2+N2+CuCl
⊕⊖ ∆
ArN2BF4+NaNO2 Cu powder Ar–NO2+NaBF4+N2

(i) Gomberg reaction (replacement by an aryl group)

ArN2Cl+PhH+NaOH Ar–Ar+N2+NaCl+H2O
Benzene Diphenyl

2 5 . 2 2 | Amines and Aromatic Compounds Containing Nitrogen

ArN2Cl+PhH+NaOH Ar-Ar+N2+NaCl+H2O
PhH+NaOH Ar-Ar+N 2+NaCl+H2O
Benzene Diphenyl
nzene Diphenyl 
 Br N2Cl+C6H6+NaOH Br + N2+NaCl+H2O
Br N2Cl+C6H6+NaOH Br + N2+NaCl+H2O

+
+ PhN2 Cl + NO2+NaOH NO2+N2+NaCl+H2O
PhN2 Cl + NO2+NaOH NO2+N2+NaCl+H2O

Whatever is the nature of the substituent in the second component, o-and p-substitutions occur and the
reaction takes place by the free radical mechanism.
 OH  
PhN2Cl Ph-N=OH Ph +N2+ OH

Ph +PhNO2+ OH

NO2+H2O

( j) Meerwein Reaction:
(i)I. PhN2Cl+CH2=CH-CN(Acrylonitrile)


(ii) 
II. Ph +CH2=CHCN PhCH2CHCN
2+
Cu
Cl
 Cl
PhCH2CHCN + Cu PhCH2CH-CN

(iii) Addition to α, β-unsaturated acid is accompanied by decarboxylation.


ArN2Cl+PhCH=CH-COOH
Cinammic acid

PhCH=CH-Ar+N2+CO2+HCl
(Stilbene)

(iv) Mechanism:

PhCH=CH-COOH ArCl
-CO2
and Ph-CH-CH-COOH
-HCl Cl Ar
Ph-CH=CH-Ar

Reduction:

Zn+HCl Zn+HCl
Ar–N=N–Cl [Ar–NH.NH2]
[H] [H]
Ar–NH2+ NH3
Chem i str y | 25.23

7.4 Coupling Reactions (Retention of Diazo Group)


 OH, pH 9-10
(a) Ph-NNCl + H OH 273-278K

Ph-N=N OH-HCl
p-Hydroxyazobenzene
(Orange)

 
H ,pH 4-5
(b) Ph-NNCl +H2N

H, 
Rearranges Ph-N=N-NH + HCl

Ph-N=N NH2
p-Amino azobenzene
(Yellow)

(c)
 NaNO2/HCl
NaO3S NH2 273-278K
Sodium salt of sulphanilic acid

N,N-Dimethyl aniline  
NaO3S NNCl
OH
273-278K

 Me
NaO3S N=N N + HCl
Me

(d) With excess of diazonium salts, the bisazo (o- and p-) and the trisazo compounds may be produced.
Ph-N=N-Cl+PhOH

PhN2Cl
OH N=N-Ph

OH N=N-Ph
PhN2Cl
N=N-Ph
Bisazo compound
OH
Ph-N=N N=N-Ph

N=N-Ph
The introduction of a second azo group is facilitated by the presence of an alkyl group in the para-position to
the hydroxyl group or by two (–OH) groups in the m-position.
2 5 . 2 4 | Amines and Aromatic Compounds Containing Nitrogen

(e) Dye test (test for 1º aromatic amine):



Coupling position
OH
273-278 K  OH
Ar-NH2+HONO+HCl Ar-NNCl + N=N
Dil. NaOH

 - Naphthol
1-Phenylazo-2-naphthol
(Orange dye)
Coupling with α -or 1-naphthol takes places at the position shown by an arrow.
OH

(Coupling position)

(f) 
In case, a compound contains both (–OH) and (–NH2) groups, the coupling takes place at o-position of the
amino group in the acid solution.
In the alkaline solution, coupling takes place at o-position to the (–OH) group.
OH OH OH
N=N-Ph
PhN2Cl PhN2Cl
Alkaline Acid
solution solution N=N-Ph
NH2 NH2 NH2

(g) 
In cases where a (–COOH) or (–SO3H) group is present at p-position to (–OH) or to (–NH2). it is an example of
ipso substitution, special case of electrophilic aromatic substitution where the leaving group is not hydrogen.

OH OH OH

PhN2Cl PhN2Cl
pH=9-10 pH=9-10

COOH N=N-Ph SO3H

NH2 NH2 NH2


PhN2Cl PhN2Cl
pH=4-5 pH=4-5

COOH N=N-Ph SO3H


Chem i str y | 25.25

PLANCESS CONCEPTS

•• Diazonium salts of alkanes, alkenes and alkynes are not at all stable at room temperature while benzene
Diazonium salts are stable to some extent due to delocalization of the positive charge in the benzene ring.
•• Whenever you see a nucleophile with benzene Diazonium salt, substitute it in place of N2, as it is a very
good leaving group.
•• Benzene Diazonium salt gives an orange red dye with β -naphthol due to extended conjugation and is
hence a test for it.
•• Being an electron deficient species benzene Diazonium salt shows a coupling reaction with an electron rich
species such as phenol and amines.
Nikhil Khandelwal (JEE 2009 AIR 94)

Illustration 11: Give the decreasing order of the reactivity of the diazonium ion coupling with the phenol. 
 (JEE MAIN)
⊕ ⊕ ⊕ ⊕ ⊕
(I) p-NO2–C6H4 N2
(II) p-Cl–C6H4 N2 (III) C6C5 N2 (IV) p-Me–C6H4 N2 (V) p-MeO–C6H4 N2

Sol: The more Electron withdrawing group in diazonium ion, the faster the coupling is.
The more EWG in diazonium ion, the faster the coupling is.
(I) > (II) > (III) > (IV) > (V) (I) ⇒ (p-NO2, –I, and –R), (II) ⇒ (p-Cl, –I)
(III) ⇒ Standard (IV) ⇒ (p-Me, +I, and H.C.)
(V) ⇒ [p-MeO–, –I, and + R, net ED power of Me–O is greater than ED power of (Me–) group
Illustration 12: Starting from benzene or toluene or aniline and with the aid of the diazonium salt synthesise the
following: p-Nitrobenzene (JEE MAIN)

Sol: We are going to prepare p-Nitrobenzene from Aniline. Since, –NH2 is o- and NH2 NO2
p- directing, introduce one (–NO2) group directly by nitration after protecting
(–NH2) group (direct nitration of aniline would yield mixture and tarry products). (a)
Nitration is done using the nitrating mixture (conc.HNO3+conc.H2SO4).Next step is
deprotection which is done by using an aqueous base. Now we can introduce the
NO2
second (NO2) group via diazonium salt method.

NH2 NHAc NHAc NH2

Ac2O HNO3 OH/H2O


+H2SO4

NO2 NO2
Major

NO2 [NN]Cl

NaNO2 NaNO2
+Cu2O +HCl
0-5
2 5 . 2 6 | Amines and Aromatic Compounds Containing Nitrogen

Illustration 13: Explain why 2,4-dinitrobenzene diazonium ion couples with anisole but Ph N 2⊕ does not. Write the
coupling reaction. (JEE ADVANCED)

Sol: The ring is not sufficiently activated by –OMe group for it does not
NN+H

:
O2N OMe
react with most Ph N 2⊕ However, e -withdrawing (–NO2) groups make
this diazonium ion less stable and thus more reactive than Ph N 2⊕ . NO2 Anisole

The ring is not sufficiently activated by –OMe group for it does not
react with most Ph N 2⊕ . However, e -withdrawing (–NO2) groups make O2N NN OMe
this diazonium ion less stable and thus more reactive than Ph N 2⊕ .
NO2

Illustration 14: Convert the following: (JEE MAIN)

Aniline H2N SO2NH2


Sulphanilamide

O O
NH3
Sol: PhNH2 Ac2O
PhNHAc
HO-SO2-Cl
Cl S NHAc H2 N S NHAc 1. H3O
Chlorosulphonic
2. OH
acid
O O
O
H2N S NH2

POINTS TO REMEMBER

Preparation of amines

Nucleophilic Reduction of nitrogen Rearrangement


Reductive amination
Substitution containing reactions
functional groups

Hoffmann’s Claisen-Schmidt
Ammomolysis From
- NO₂ (nitro)
compounds Lossen
Gabriel
Phthalimide From
Hoffmann
nitrites
bromamide

From
amides Curtius

From
Ritter
others
Chem i str y | 25.27

Solved Examples

JEE Main/Boards HNO2


3 4
OH 5 OH
2 
Example 1: Distinguish between the following pairs: NH2 CH2
1
(I) (II)

a. (PhNH3 )2+ SO24− and H3N SO3 3
4

-H 5
O OH
 ⊕  2
b. Me4NΘClΘ and Me3 NH ClO– 1
  Cyclopentanone
(X)
Sol: First two compound can be differentiated by using
BaCl2 solution as barium will form a ppt with sulphate
ion. Other two compound as can be distinguished by Example 4: + I-N=C=O (VIII)
H2 O
(IX)
treated with NaOH.
a. Add BaCl2 solution. (I) is a sulphate salt and will
give a white precipitate of BaSO4. (II) is a sulphonate Sol: First step will lead to formation of an isocyanate.
(sulphonic acid salt) and will not give any precipitate. On treatment with water it yields primary amine.

b. Add concentrated NaOH and heat the mixture. (II will 


change to a volatile free base Me3N. which has typical + I-N=C=O (I N=C=O)
NH3 odour. Compound (I) will not react.
I I
H2 O
Example 2: Complete the following reactions: NH2 N=C=O
Zn+ aq. NH Cl
(VIII)
4 → (I)
a. PhNO2  (IX) 2-Iodocyclopentyl
isocyanate
Al O /aq. NaOH
b. p-Me–C6H4NO2 
2 3 → (II)
Example 5: Give the reagents in the following reactions:
Sol: a. (I) PhNHOH (N-Phenyl hydroxylamine)

 Me Me Me
b. (II) Me N=N Me
Br Br
(1)? (2)?
O
p-Methyl azoxy benzene (3)?
(4)?
NO2 NO2 Br
Example 3:

2. Sn + HCl;OH
HNO2
OH (X) Sol: 1. Br2 + Fe
NH2 3. HNO2 + 0-5ºC 4. CuBr

Sol: It is an example of pinacole-pinacolone type ring Example 6:


expansion which takes place via carbocation.
Cl
AlCl3 1. HNO3/H2SO4
PhH+Me (C) (D)
2. Sn/HCl
(A) Me
3.OH
(B)
2 5 . 2 8 | Amines and Aromatic Compounds Containing Nitrogen

Sol: First step is Friedal craft alkylation. This step Example 8: Identify compounds (A) to (E) in the
produces a new chiral centre on the product but it is following:
optically inactive. The alkylated product is subjected
Sn/HCl 1. HNO3, 5C
to nitration to introduce nitro group at para position p-NO2C6H4OEt (A)
2. PhOH
(B)
on treatment with reducing agents like Sn+HCl, nitro
group gets reduced to amino group. MeCOCl SnCl2 OH
(E) (D) (C)
Me
*
Et2SO4
Me (C10H13O2N)
Me
HNO3+ Sn+HCl
(C) Ph Me Sol:
H2SO4
OH

NO2 OEt OEt


()or Racemate
Sn/HCl 1. NaNO2, 5C
Me
* Me 2. PhOH

NO2 NH2
(A)
NH2
()or Racemate EtO N=N OH
(D) (B)
OH+Et2SO4
Example 7: Explain the formation of the mixture
PhCH2CHO (I) and PhCOMe (II) when PhCH(OH)
CH2NH2(A) is treated with HNO2. EtO N=N OH
(C)
Sol: Attack of Nitrous acid produces primary carbocation
[H] SnCl2
which gets rearranged to secondary carbocation and
tertiary carbocation. Due to formation of two different
carbocation we get different product. With secondary 2EtO NH2
carbocation we end up getting PhCH2CHO and with
tertiary carbocation we get PhCOMe. (D)

OH MeCOCl
OH
NH2
HNO2 1,2-H
Ph Ph CH2 Shift 2EtO NH COMe
(A) 1,2-Ph
shift
(E)
OH Phenacetin
OH (Analgesic and antiopyretic)
-H Ph CH3
H2 O
CH2-Ph H2O -H
O=HC-H2C-Ph
O
(I)

Ph CH3
Chem i str y | 25.29

Example 9: Convert benzene to o-nitro aniline as the amine on treatment with benzene sulphonyl chloride
only product. its derivative.

Sol: 
(A) Ph-NH2 Cl (B) ⇒ Ph—NH—CH3
+
NO2
NO2 NH2
HNO3/H2SO4 Sn/HCl CH3

CH3COCl Base
O
NH O NH O (C) Ph N S Ph (insoluble in NaOH)
Conc.
H2SO4 CH3 O
CH3 CH3
SO3H

HNO3/H2SO4
O CH3 Example 2: An organic compound (A) of molecular
weight 135 on boiling with NaOH evolves a gas which
NH O NH
Steam gives dense white fumes on bringing a rod dipped
in HCl near it. The alkaline solution thus obtained on
CH3
SO3H NO2 NO2 acidification gives the precipitate of a compound (B),
H 3O having molecular weight 136. Treatment of (A) with
Hydrolysis HNO2 also yields (B), whereas its treatment with Br2/
KOH gives (C). Compound (C) reacts with cold HNO2 to
NH2
give (D) which gives red colour with ceric ammonium
nitrate. On the other hand, (E) an isomer of (A) on boiling
NO2 with dilute HCl gives an acid (F), having molecular
weight 136. On oxidation, followed by heating, (F) gives
an anhydride (G) which condenses with benzene in the
Example 10: m-Me–C6H4NO2 
LAH
→ (III) presence of AlCl3 to give anthraquinone, Give structures
of (A) to (G) with proper reasoning.
Sol: (III) is
Sol:
:
:

Me N=N O
(Anti) NaOH
Ph CH2 C NH2 PhCH2COONa NH3

Me p-Methyl (A)
and
azobenzene Mw=135
HNO2 PhCH2COOH
Me N=N Me
Br2-KOH NH2 -OH (B)
(Syn) Mw=136
Cold
PhCH2NH2 PhCH2OH
IINO2
(C) (D)
JEE Advanced/Boards (Test for alcohol)
(Positive ceric ammonium)
nitrate test
Example 1: The aqueous solution of a nitrogen and
chlorine containing organic compound (A) is acidic O
to litmus. (A) on treatment with aqueous NaOH, it HCl
C NH2 COOH(Mw=136)
gives a compound (B) containing nitrogen, but not
chlorine. Compound (B) on treatment with C6H5SO2Cl CH3 CH3
in the presence of NaOH gives an insoluble product (C) (E) (F)
C13H13NO2S. Give the structures of (A) and (B).
Oxidation
O
Sol: Quaternary ammonium salt on treatment with
aqueous NaOH gives secondary amine. Secondary 
C COOH
H2 O
C O COOH
(G)
O [H] SnCl2
HCl
NH2 and Aromatic Compounds
2 5 . 3 0 | CAmines COOH(Mw=136)
Containing Nitrogen
2EtO NH2
CH3 CH3
(E) (F) (D)
Oxidation MeCOCl
O

C COOH 2EtO NH COMe
H2 O
C O COOH (E)
(G) Phenacetin
O (Analgesic and antipyretic)

O O

AlCl3 Example 4:
+O
F.C. Me Me
H OC Me
OH
O (1)? (2)?
H2O H2SO4
(3)?
Br Br Br Br
O NH2 NH2

Sol:
1. Br2 + Fe
O 2. HNO2, 0-5ºC
Anthraquinone
3. H3PO2

Example 3: Identify compounds (A) to (E) in the following: Example 5:


Me Me
1.HNO2 .5o C
Sn/HCl
P − NO2C6H4 OEt 
→ (A)  → NO2 NH2
2.PhOH (1)? (2)?
Θ
OH → (C) →
2 SnCl
MeCOCl
(B)  (D)  →(E)(C10 H13O2N) Me Me
Et2SO 4
NHAC NHAC
(3)?
OEt OEt
O2N
1. NaNO2, 5C (4)?
Sn/HCl
2. PhOH Me Me
NH2 NH2
NO2 NH2 (5)?
(A) O2N (6)?
O2N Br
(7)?
EtO N=N OH
(B) Me Me
Br Br
OH+Et2SO4

(8)?
Br O2N Br
(9)?
EtO N=N OEt (10)?
(C)
[H] SnCl2
Sol: 1. Sn + HCl 2. Ac2O
O

3. HNO3 + H2SO4 4. OH
2EtO NH2

(D)
MeCOCl
Chem i str y | 25.31

5. Br2 + Fe 6. HNO2, 0-5ºC Sol:


O

i. Compound (A) is insoluble in water and burns with a
7. CuBr 8. Sn + HCl; OH
smoky flame; hence it should be an aromatic compound.
9. HNO2, 0-5ºC 10. H3PO2 ii. It has no specific element such as N, S and halogens
and gives a CO2 gas with NaHCO3 solution; hence it
Example 6: should contain (–COOH) group.
iii. It gives the oil of wintergreen (methyl salicylate) with
Me CH3OH in acidic medium; hence it is salicylic acid.
O2N C Cl OH O OH O
E2NH + (C)
O
(A) (B) OH CH3OH/H OCH3
H2+Ni
(E) Room temp.
(D)
Compound (A) Methyl salicylate
Novocaine
(Local anaeshetic)
iv. Compound (B) is water soluble and burns with a
Sol: non-smoky flame; hence it should be an aliphatic
compound.
v. This compound has less carbon content because its
:

E2NH +
O sodium extract is prepared with sucrose and it gives a
(A) Prussian blue colour with the freshly prepared solution
(B)
of FeSO4 + 2-3 drops of NaOH and a few drops of
Et2N O H Cl C NO2 H2SO4; hence it is a nitrogen-containing compound.
(C) O NaCN+FeSO4 → Na4[Fe(CN)6]+Na2SO4
[O]
Et2N H2C H2C O C NO2
FeSO4  → Fe2(SO4)3
H2+Ni O
3Na4 [Fe(CN)6 ] + 3Fe2 (SO 4 )3 → Fe4 [Fe(CN)6 ]3 + 6Na2SO 4
(Pr ussian blue colour)
Et2N (CH2)2 O C NH2
O
vi. On heating, it gives ammonia gas, which turns red
(D)
litmus blue. Hence, it contains (-CONH2) group.
Example 7: A mixture of two organic compounds is
vii. It also gives the biuret test.
added to cold water. After filtration, water-insoluble
compound (A) burns with a smoky flame and it does ∆
2NH2CONH2  → NH2CONHCONH2 + NH3 ↑
not respond to Lassaigne’s and Beilsteins test. When Biuret
a small amount of this is added to NaHCO3 solution, a Cu2 +
colourless gas is evolved with effervescence. When this NH2CONHCONH2 → Violet colour
compound is heated with CH3OH in acidic medium, it
gives the characteristic smell of the oil of wintergreen. Hence, this compound is urea (NH2CONH2).
Compound (B), which is water soluble, burns with a
non-smoky flame and its sodium extract is prepared Example 8: Give the structural formula of a chiral
with cane sugar. It gives a Prussian blue colour with compound C8H11N (X), which dissolves in dilute HCl and
freshly prepared solution of FeSO4 + 2-3 drops NaOH evolves N2 gas with HNO2.
and with few drops of H2SO4. When a small amount of (2nc + 2) − (nH − nN ) 18 − 10
this compound is heated in a dry test tube, a colourless Sol: D.U. in (X) = = = 4º
2 2
gas is evolved that turns moist red litmus paper i. 4 D.U. is (X) suggests that is contains benzene ring.
blue and a white residue is left. This white residue is
dissolved in water and a drop of CuSO4 is added in the ii. (X) is a 1º amine since it dissolves in HCl and gives N2
basic medium-a violet colour is obtained. Identify the with HNO2.
compounds (A) and (B) with the help of the reactions iii. The remaining C atom and the NH2 must form a
involved. chiral molecule. So, (X) is:
2 5 . 3 2 | Amines and Aromatic Compounds Containing Nitrogen

Me

Ph * NH2

Example 9: Identify compounds (A) through (E) in the


following:

1. OH Zn/HCl
p-NO2C6H4OH 
2. EtBr
→ (A)  → (B)

NaNO /HClPhOH LiAlH


2
→
5ºC
(C) → (D) 
4 → (E)+(F)

(F) dissolves in NaOH.

Sol:
OH ONa OEt OEt OEt OEt

OH EtBr Zn/HCl NaNO2+HCl at 5C


(or) (C)
NaOH


NO2 NO2 NO2 NH2 [NH3]Cl NN-Cl
(A) PhOH
(B) Coupling

OH OEt OEt

LiAlH4
+

NH2 NH2 NN OH


(F) (E)
(D)
(Soluble in NaOH)

JEE Main/Boards

Exercise 1 Q.2 Describe a method for the identification of primary,


secondary and tertiary amines. Also write chemical
Q.1 Arrange the following: equations of the reactions involved.

(i) In decreasing order of the pKb values:


Q.3 Explain Hofmann Bromamide reaction with
C2H5NH2, C6H5NHCH3, (C2H5)NH and C6H5NH2 Mechanism.
(ii) In increasing order of basic strength.
Q.4 Why cannot aromatic primary amines be prepared
C6H5NH2, C6H5N(CH3)2, (C2H5)2NH and CH3NH2
by Gabriel phthalimide synthesis ?
(iii) In increasing order of basic strength:
(a) Aniline, p-nitroaniline and p-toluidine Q.5 Write the reactions of (i) aromatic and (ii) aliphatic
primary amines with nitrous acid.
(b) C6H5NH2, C6H5NHCH3, C6H5CH2NH2,
Chem i str y | 25.33

Q.6 Write one chemical reaction each to illustrate the Q.15 Write the method of formation of benzene
following diazonium chloride
(i) Hofmann Bromamide reaction.
Q.16 Account for the following:
(ii) Gabriel Phthalimide reaction
(i) Diazonium salts of aromatic amines are more stable
than those of aliphatic amines.
Q.7 Assign a reason for the following statements
(ii) Gabriel phthalimide synthesis is preferred for
(a) Alkylamines are stronger bases than arylamines.
synthesizing primary amines.
(b) How would you convert methylamine into
ethylamine?
Q.17 How will you convert:
(i) Ethanoic acid into methanamine
Q.8 Illustrate the following with an example of reaction
in each case: (ii) Hexanenitrile into 1-aminopentane
(i) Sandmeyer reaction (iii) Methanol to ethanoic acid
(ii) Coupling reaction (iv) Ethanamine into methanamine
(v) Ethanoic acid into propanoic acid
Q.9 Write the chemical reaction equations for one
(vi) Methanamine into ethanamine
example each of the following
(vii) Nitromethane into dimethylamine
(i) A coupling reaction
(viii) Propanoic acid into ethanoic acid?
(ii) Hofmann’s bromamide reaction
(iii) Aryl cyanides cannot be formed by the reaction of
Q.18 Write short notes on the following:
aryl halides and sodium cyanide.
(i) Coupling reaction
Q.10 Account for the following: (ii) Ammonolysis
(i) Aniline is weaker base than methylamine. (iii) Acetylation
(ii) Aryl cyanides cannot be formed by the reaction of (iv) Gabriel phthalimide syntheisis
aryl halides and sodium cyanide.
Q.19 Accomplish the following conversions
Q.11 Describe tests to distinguish between: Secondary
(i) Nitobenzene to benzoic acid
amine and tertiary amine.
(ii) Benzene to m-bromophenol
Q.12 Account for the following observations: (iii) Benzoic acid to aniline
(i) pKb for aniline is more than that for methylamine. (iv) Aniline to 2,4,6-tribromofluorobenzene
(ii) Methylamine solution in water reacts with ferric (v) Benzyl chloride to 2-phenylethanamine
chloride solution to give a precipitate of ferric hydroxide.
(vi) Chlorobenzene to p-chloroaniline
(iii) Aniline does not undergo Friedel Crafts reaction.
(vii) Aniline to p-bromoaniline
(viii) Benzamide to toluene
Q.13 State the reactions and reaction conditions for the
following conversion (ix) Aniline to benzyl alcohol.
(i) Benzene diazonium chloride to nitrobenzene.
Q.20 Write the equation of Curtius reaction with
(ii) Aniline to benzene diazonium chloride.
mechanism?
(iii) Ethyl amide to methylamine.
Q.21 Complete the following reactions:
Q.14 Write the physical property of aniline
(i) C6H5NH2 + CHCl3+alc. KOH →
2 5 . 3 4 | Amines and Aromatic Compounds Containing Nitrogen

(ii) C6H5N2Cl + H3PO2 + H2O → Q.2 Examine the following two structures for the
anilinium ion and choose the correct statement from
(iii) C6H5NH2 + H2SO4 (conc.) →
the ones given below.
(iv) C6H5N2Cl + C2H5OH → +
NH3 NH3
(v) C6H5NH2 + Br2(aq) →
+
(vi) C6H5NH2 + (CH3CO)2O →

(A) II is not an acceptable canonical structure, because


Q.22 Give possible explanation for each of the following:
carbonium ions are less stable than ammonium ions
(i) Why are amines less acidic than alcohols of
(B) II is not an acceptable canonical structure, because
comparable molecular masses?
it is non aromatic
(ii) Why do primary amines have higher boiling point
(C) II is not an acceptable canonical structure, because
than tertiary amines?
the nitrogen has 10 valence electrons
(iii) Why are aliphatic amines stronger bases than
(D) II is an acceptable canonical structure.
aromatic amines?

Q.3 The correct order of basic strength in CCl4


Q.23 Write the reaction and conditions for the following
conversions (1) NH3 (2) RNH2 (3) R2NH (4) R3N
(i) Aniline to benzene where R is CH3 group is
(ii) Methylamine to methyl cyanide (A) 3>2>1>4 (B) 2>3>4>1
(iii) Propanenitrile to ethylamine (C) 3>2>4>1 (D) None of these
(iv) m-Bromoaniline to m-bromophenol
Q.4 Place the following in the decreasing order of
(v) Nitrobenzene to 2,4,6-tribromoaniline.
basicity.
(1) Ethylamine (2) 2-aminoethanol
Q.24 Write the method of formation of zwitter ion?
(3) 3-aminopropan-1-ol
Q.25 Explain nitration of aniline? (A) 1>3>2 (B) 1>2>3
(C) 2>1>3 (D) None of these
Q.26 Why aniline does not give Friedel-Crafts reaction?
Q.5 Which of the following will give a positive
Q.27 How will you convert 4-nitrotoluene to carbylamine test?
2-Bromobenzoic acid?
(A) H3CNH2 (B) H3C–NH–CH3
Q.28 Draw the structure of trimethylamine and tell the (C) (CH3)3N (D) C6H5NH2
shape of the molecule. Show the angle between two
methyl groups. Q.6 Isopropylamine can be obtained by
LiAlH
(A) (CH3)2CHO + NH2OH → ? 
4→
Exercise 2
H /Ni
(B) (CH3)2CHO + NH3 → ? 
2 →
Single Correct Choice Type ∆

(C) CHOH + NH3 →


Q.1 When aniline is treated with fuming sulphuric acid
(D) All of these
at 475K, it gives
(A) Sulphanilic acid
Q.7 The basic strength of amines (ethyl) and ammonia
(B) Aniline sulphate in H2O is
(C) o-aminobenzenesulphonic acid (A) NH3>p>s>t (B) P>s>t>NH3
(D) m-aminobenzenesulphonic acid (C) s>p>t>NH3 (D) None of these
Chem i str y | 25.35

Q.8 Which of the following will have highest Kb value? Q.13 Identify compound (A) in the following oxidation
reaction.
(A) (B)
K2Cr2O7 H2SO4
N (A) O O
N
H
(C) (A) NH2 (B) OH
NH2 (D) NH2

Me Cl NH2 NH2

(C) OH (D) All of these


Q.9 The product not obtained in the following reaction,
CH3–NO2+Cl2 + NaOH → is
(A) ClCH2NO2 (B) Cl2CHNO2
(C) Cl3CNO2 (D) CH3NH2 OH

Q.10 A sequential reaction may be performed as


represented below: Q.14 NH2

SO Cl
+phosgene X. Here X is
(a) R–CH2CO2H 
2 2 → R–CH COCl
(1) 2

(A) O (B) N = C = O
NH3

(2)
→ R–CH2CONH2  → R–CH2NH 
(4)
→ NH -C -Cl
(3)

R–CH2OH  → R–CO2H
(5) (C) Cl (D) None of these
- -

The appropriate reagent for step (3) is CH- C -H


(A) NaBr (B) Bromine+alkali (C) HBr (D) P2C5 Cl

Q.11 Which of the following amine form N-nitroso Q.15 Ethylamine undergoes oxidation in the presence
derivative when treated with NaNO2 and HCl? of KMnO4 to give
(A) CH3COOH (B) CH3CH2OH
(A) H3C NH2 (B) NH2
(C) CH3CHO (D) N-oxide
(C) N(CH3)H2 (D) NH(CH3)
Q.16 Baker Mulliken’s test is used to detect the presence

Q.12 The strongest base among the following is (A) –COOH gp (B) –NO2
(C) –OH (D) –NH2
H2NH2N H2NH2N H2NH2N H2NH2N

(A) (A) C =CNH = 2NH2(C) (C) C =CO= O (D)Q.17
= NH (B) (B)C =CNH (D)CH
t-amines
= OH
CH = OHwith different alkyl group has a chiral
H2NH2N H2NH2N H2NH2N nitrogen
H2NH2N atom still it is optically inactive because
(A) Chiral N-atoms cannot rotate plane polarized light
N H2N H2N H2N H2N H2N
(C2) (C)C = OC = (D)
C = NH O (D) (B) The lone pair prevents the rotation of plane polarized
(B)
C = NH CH = CH
OH = OH
2
light
N H 2N H N H2N H N H2N
(C) Both of these
2 2

(D) None of these


2 5 . 3 6 | Amines and Aromatic Compounds Containing Nitrogen

Q.18 In CH3NO2 we can observe


List I List II
(A) H-bonding
(iv) CH3CH(OH)CH3 (s) With Lucas reagent
(B) α-halogenation reaction cloudiness appears after 5
minutes
(C) Tautomerism
(D) All of these (A) (i) - q, (ii) - p, (iii) - s, (iv) - r
(B) (i) - r, (ii) - q, (iii) - p, (iv) - s
Q.19 Match list I (condition of reaction of nitrobenzene) (C) (i) - q, (ii) - r, (iii) - p, (iv) - s
with list II (products formed) and select the correct
answer the codes given below. (D) (i) - s, (ii) - q, (iii) - r, (iv) - p

List I List II Q.2 A primary amine is formed an amide by the


(i) Sn and HCl (p) Hydrazobenzene treatment of bromine and alkali. The primary amine has
 (2004)
(ii) Zn and NH4Cl (q) Azoxybenzene
(A) 1 carbon atom less than amide
(iii) Methanolic NaOMe (r) Phenyl hydroxylamine
(B) 1 carbon atom more than amide
(iv) Zn and KOH (s) Aniline
(C) 1 hydrogen atom less than amide
(A) (i) - q, (ii) - p, (iii) - r, (iv) - s
(D) 1 hydrogen atom more than amide
(B) (i) - s, (ii) - r, (iii) - q, (iv) - p
(C) (i) - p, (ii) - s, (iii) - q, (iv) - r Q.3 Indicate which nitrogen compound amongst the
following would undergo Hofmann’s reaction (i.e.
(D) (i) - p, (ii) - r, (iii) - q, (iv) - s
reaction with Br2 and strong KOH) to furnish the primary
amine (R–NH2)  (1989)
Q.20 The increasing order of basicity of RCN, RCH=NR
and RNH2 is (A) (B)
(A) RCN<RCH=NR<RH2N
(C) (D)
(B) RNH2<RCN<RCH=NR
(C) RCN >RCH=NR <RNH2
(D) None of these Q.4 The order of basic strength among the following
amines in benzene solution is (1991)

Q.21 How many isomeric amines with that formula (A) CH3NH2>(CH3)3N>(CH3)2NH
C7H9N contain a benzene ring? (B) (CH3)2NH>CH3NH2 >(CH3)3N
(A) Two (B) Three (C) Four (D) Five (C) CH3NH2>(CH3)2NH>(CH3)2NH
(D) (CH3)3N>CH3NH2>(CH3)2NH
Previous Years' Questions
Q.5 The refluxing of (CH3)2NCOCH3 with acid gives
Q.1 Match the compounds given in list I with their  (1996)
characteristic reactions given in list II. Select the correct (A) 2CH3NH2 + CH3COOH
option (2010)
(B) 2CH3OH + CH3COOH
List I List II (C) (CH3)2NH + CH3COOH
(i) CH3CH2CH2CH2NH2 (p) Alkaline hydrolysis (D) (CH3)2NCOOH + CH4
(ii) CH3C≡CH (q) With KOH (alcohol) and
CHCl3 produces bad smell
(iii) CH3CH2COOCH3 (r) Gives white ppt. with
ammonical AgNO3
Chem i str y | 25.37

Q.6 Order of basicity of ethyl amines is (1988) Q.13 The electrophile, E+ attacks the benzene ring to
generate the intermediate σ -complex of the following,
(A) Secondary > Primary > Tertiary
which σ-complex is of lowest energy?  (2008)
(B) Primary > Secondary > Tertiary
NO2
(C) Secondary > Tertiary > Primary
H
(D) Tertiary > Primary > Secondary
(A) + (B) + E
Q.7 The following reaction is RX+KOH (solid) → heat

 (1988)
(A) Nucleophilic substitution H E
(B) Electrophilic substitution NO2 NO2

(C) Free radical substitution H

(D) None of these (C) + E (D) + H

E
Read the assertion and reason carefully to mark the
correct option out of the options given below:
Q.14 In the chemical reactions the compounds ‘A’ and
(a) If both assertion and reason are true and the reason
‘B’ respectively are  (2010)
is the correct explanation of assertion.
(b) If both assertion and reason are true but reason is NH2
not the correct explanation of the assertion.
NaNO2 HBF4
(c) If assertion is true but reason is false. A E
HCl, 278 K
(d) If assertion is false but reason is true.

(A) Nitrobenzene and fluorobenzene


Q.8 Assertion: Benzene diazonium chloride does not
give tests for nitrogen. (B) Phenol and benzene

Reason: N2 gas lost during heating  (1999) (C) Benzene diazonium chloride and fluorobenzene
(D) Nitrobenzene and chlorobenzene
Q.9 Assertion: Amines are basic in nature.
Reason: Presence of lone pair of electron on nitrogen Q.15 Which of the following compounds can be
atom. (1999) detected by Molisch’s test?  (2012)
(A) Nitro compounds (B) Sugars
Q.10 Assertion: Alkyl isocyanides in acidified water
(C) Amines (D) Primary alcohols
give alkyl formamides.
Reason: In isocyanides, carbon first acts as a nucleophile Q.16 On heating an aliphatic primary amine with
then as an electrophile.  (2005) chloroform and ethanolic potassium hydroxide, the
organic compound formed is:  (2014)
Q.11 Assertion: Amines are more basic than esters and
(A) An alkanol (B) An alkanediol
ethers.
(C) An alkyl cyanide (D) An alkyl isocyanide
Reason: Nitrogen is less electronegative than oxygen.
It is in better position to accommodate the positive
charge on the proton.  (2007) Q.17 Considering the basic strength of amines in
aqueous solution, which one has the smallest pKb
value?  (2014)
Q.12 Assertion: Nitrobenzene is used as a solvent in
Friedel-Craft’s reaction. (A) (CH3)2NH (B) CH3NH2

Reason: Fusion of nitrobenzene with solid KOH gives a (C) (CH3)3N (D) C6H5NH2
low yield of a mixture of o- and p- nitro phenols.  (2008)
2 5 . 3 8 | Amines and Aromatic Compounds Containing Nitrogen

Q.18 In the reaction  (2015) COOH

NH2
(A) (B) H3C CH3
NaNO2/HCl CuCN/KCN
D  E + N2
o
0-5 C CH3

CH3
the E is: product CN CH3

(C) (D)

CH3

JEE Advanced/Boards

Exercise 1 Q.4 An organic compound (A), C6H4N2O4, is insoluble


in both dilute acid and base and its dipole, moment is
Q.1 Aspartame, an artificial sweetener, is a peptide and zero. Deduce the structure of (A).
has the following structures:
Q.5 Explain the following observations:
NH2 CH2C6H5
(i) Aniline dissolves in aqueous HCl.

HOOC—CH2CH—CONH—CH—COOCH3 (ii) The amino group in ethylamine is basic whereas that
in acetamide it is not basic.
(i) Identify the four functional groups.
(iii) Dimethylamine is a stronger base than
(ii) Write the zwitter ionic structure trimethylamine.
(iii) Write the structures of the amino acids obtained (iv) Sulphanilic acid although has acidic as well as basic
from the hydrolysis of aspartame. group, it is soluble in alkali but insoluble in mineral
(iv) Which of the two amino acids is more hydrophobic? acids.

Q.2 Compound of A(molecular formula C9H11NO gives Q.6 Explain, why?


a positive Tollen’s test and is soluble in dilute HCl. It (i) Glycine exists as H3N+CH2COO– while anthranilic acid,
gives no reaction with benzene sulphonyl chloride or p–NH2–C6H4–COOH does not exist as dipolar ion.
with NaNO2 and HCl at 0ºC. (A), upon oxidation with
(ii) Benzenesulphonic acid is a stronger acid than
KMnO4 gives an acid (B). When (B) is heated with soda-
benzoic acid.
lime, compound (C) is formed which reacts with NaNO2
and HCl at 0–5ºC. What is (A)? (iii) A weakly basic solution favours coupling with
phenol.
Q.3 An organic compound A, when treated with nitrous (iv) It is difficult to prepare pure amines by ammonolysis
acid yields an alcohol B, C4H10O with the evolution of alkyl halides.
of N2. B on careful oxidation yields a substance C of
vapour density 36 which forms oxime; B can react with
Q.7 Explain with reason?
NaHSO3 but does not reduce Fehling solution. Identify
compound A and write the structural formulae of the (i) Although trimethylamine and n-propylamine have
isomeric compounds that behave with HNO2 in the same molecular weight, the former boils at a lower
same manner. temperature (3ºC) than the latter (49ºC).
Chem i str y | 25.39

(ii) Dimethylamine is a stronger base than methylamine


(ii) P2O5 +
H
but trimethylamine is a weaker base than both CONH2 F 
G
dimethylamine and methylamine.
KOH
(iii) Silver chloride dissolves in aqueous solution of (iii) EtNH2 + KCN + Br2  → KBr +H
methylamine. Explain.
(iv) N(CH3)2 + HNO2 I
Q.8 Explain it?
(i) An aqueous solution of ethylamine gives a red (v) 2,4-Dinitroaniline →2 (i) NaNO /HCl, 5ºC
(J)
(ii) anisole
precipitate with ferric chloride. Explain.
NaOH
(ii) Tertiary amines do not undergo acetylation. (vi) C6H6 
Oleum
→ (K)  → (L)
Comment NaOH

heat
→ (M)

(iii) 2,6-Dimethyl-N,N-dimethylaniline, although has
a free p-position, does not undergo coupling with SO3H OH
benezene diazonium chloride. Comment.
(iv) In the following compounds: (vii) I CHCl3/NaOH
N
O

(viii) SO3H
Fuming
O
(i)NaOH fuse
+ P
H2SO4 (ii)H
N N
Et SO HCN, HCl
H N H 2 4 → (Q) →
(ix) Phenol 
NaOH AlCl 3
PhNH.NH2
(I) (II) (III) (R)  → (S)

(x) CH3CONHC6H5 


Br2 , Fe
→T + U
N
(A)
(xi) C6H5N2Cl  → (V) Gattermann reaction
H

(IV) Q.11 Give structures for the compounds (A) to (I):


The order of basicity is I > III > II > IV. Explain. NaNO2/HCl KCN
C8H11N B CuCN C
(A) Hot H2SO4

Q.9 Explain it with reason. Heat to Hot


I G D
(i) tert-Butylamine cannot be prepared by the action of m.p. aq.KMnO4
Cl2, 2 moles
NH3 on tert-butyl bromide. CH3OH
u.v.
at 30C Hot
(ii) Isocyanides are hydrolysed by dilute acids but not Hot KMnO4
by alkalis to form amine and formic acid. H2SO4 Hot NaOH

(iii) How will you explain the acidic nature of 1º and 2º H F acidify
nitroalkanes?
(iv) Aniline does not undergo Friedel Craft’s reaction? Q.12 When 2.25 g of an unknown amine was treated
(v) Although boron trifluoride adds on trimethylamine, with nitrous acid, the evolved nitrogen, corrected to
it does not add on triphenylamine. Comment. S.T.P. measured 560 ml. The alcohol isolated from the
reaction mixture gave a positive iodoform reaction.
What is the structural formula of the unknown amine?
Q.10 Complete the following reactions:

(i) C6H5COOH 


PCl
5 → [C] 
3 → [D]NH Q.13 The aqueous solution of a nitrogen and chlorine
containing organic compound (A) is acidic towards litmus.

P O
2 5 → C H CN 
 → [E]
2  H /Hi
(A) on treatment with aqueous NaOH gives a compound
6 5
(B), containing nitrogen, but not chlorine. Compound (B)
2 5 . 4 0 | Amines and Aromatic Compounds Containing Nitrogen

on treatment with C6H5SO2Cl in the presence of NaOH Q.22 An optically active amine (A) is subjected to
gives an insoluble product (C), C13H13NO2S. Give the exhaustive methylation and Hofmann elimination to
structures of compounds (A) and (B). yield an alkene (B). (B) on ozonolysis gives an equimolar
mixture of formaldehyde and butanal. Deduce the
Q.14 An organic compound (A) composed of C, H and structures of (A) and (B). Is there any structural isomer
O gives characteristic colour with ceric ammonium to (A), if yes draw its structure.
nitrate. Treatment of (A) with PCl5 gives (B), which reacts
with KCN to form (C). the reduction of (C) with warm Q.23 An aromatic compound (a) having molecular formula
Na/C2H5OH products (D), which on heating gives (E) C7H7NO2 dissolves in NaHCO3 to evolve CO2 and when
with evolution of ammonia Pyridine is obtained on reacted with NaNO2/HCl forms (b), C7H6O3. (B) dissolves in
treatment of (E) with nitrobenzene. Give structure of NaHCO3 and gives colour reaction with FeCl3 and can be
compounds (A) to (E) with proper reasoning. prepared by the action of CCl4 and NaOH on phenol. When
(B) is reacted with excess HNO3, it forms (C), C6H3N3O7.
Q.15 One mole of bromo derivative (A) and mole of (C) undergoes acetylation and decomposes NaHCO3 to
NH3 react to give one mole of an organic compound evolve CO2. On reaction with PCl5 (C) is converted to (D),
(B). (B) reacts with CH3I to give (C). Both (B) and (C) react C6H5N3O6Cl which when reacted with water gives back (C).
with HNO2 to give compounds, (D) and (E) respectively. Identify compounds (A) to (D).
(D) on oxidation and subsequent decarboxylation
gives 2-methoxy-2-methyl propane. Give structures of Q.24 Compound (A) having M.F. C8H8O on treatment
compounds (A) to (E) with proper reasoning. with NH2OH.HCl gives (B) and (C). (B) and (C) rearrange
to give (D) and (E), respectively on treatment with acid.
Q.16 What happens when cyclopentanone reacts with Compounds (B), (C), (D) and (E) are all isomers of molecular
formula C8H9NO. When (D) is boiled with alcoholic KOH,
(i) CH3CH2NH2 (1º amine) and oil (F) C6H7N separated out. (F) reacts rapidly with
(ii) (CH3CH2)2NH (2ºamine) CH3COCl to give back (D). On the other hand (E) on boiling
with alkali followed by acidification gives a white solid (G),
C7H6O2. Identify the compounds (A) to (G).
Q.17 Cyclohexyl amine is a stronger base than aniline.
Why?
Q.25 An aromatic compound (A), having M.F
C7H5NO2Cl2 on reduction with Sn/HCl gives (B), which
Q.18 How does the formation of 2º and 3º amines can
on reaction with NaNO2/HCl gives (C). Compound (B)
be avoided during the preparation of 1º amines by
is unable to form a dye with β -naphthol. However, (C)
alkylation?
gives red colour with ceric ammonium nitrate and on
oxidation gives an acid (D), having equivalent weight
Q.19 It is necessary to acetylate aniline first for 191. Decarboxylation of (D) gives (e) which forms a
preparing bromoaniline. Why? single mononitro derivative (F), on nitration. Give the
structures of (A) to (F) with proper reasoning.
Q.20 Dimethyl amine is a stronger base than
methylamine but trimethylamine is a weaker base than Q.26 An organic compound (A) of molecular weight
both dimethyl amine and methylamine. Why? 135, on boiling with NaOH evolves a gas which gives
white dense fumes on bringing a rod dipped in HCl near
Q.21 From analysis and molecular weight it. The alkaline solution thus obtained on acidification
determination, the molecular formula of (A) is C3H7NO. gives the precipitate of a compound (B) having
The compound gave following reactions. molecular weight 136. Treatment of (A) with HNO2 also
yields (B), whereas its treatment with Br2/KOH gives
(i) On hydrolysis, it gives an amine (B) and a carboxylic (C). Compound (C) reacts with cold HNO2 to gives (D),
acid (C) which give red colour with ceric ammonium nitrate.
(ii) Amine (B) reacts with benzene sulphonyl chloride On the other hand, (E) an isomer of (A) on boiling with
and gives a product which is insoluble in aqueous dilute HCl gives an acid(F), having molecular weight
sodium hydroxide solution. 136. On oxidation followed by heating, (F) gives an
anhydride (G), which condenses with benzene in the
(iii) Acid (C) on reaction with Tollen’s reagent gives a presence of anhydrous AlCl3 to give anthraquinone.
silver mirror when are A, B and C. Explain the reactions. Give the structures of (A) to (G) with proper reasoning.
Chem i str y | 25.41

Q.27 An organic compound (A) having M.F C7H9N on Exercise 2


treatment with NaNO2 and HCl at room temperature
forms another compound (B), C7H8O. When (A) or Single Correct Choice Type
(B) is treated with bromine water, they form dibromo
derivatives, When (A) is reacted with chloroform and Q.1 Match the compounds in list I with the appropriate
alkali, it forms (C) having the molecular formula C8H7N. test that will be answered by each one of them in list II
Hydrolysis of (C) followed by reaction with NaNO2 from the combinations shown.
and HCl at low temperature and subsequent reaction
with HCN in the presence of Cu (D), which is isomeric Selects the correct answer using the codes given below
to (C). (D) on hydrolysis followed by oxidation gives the list.
a dibasic acid which on halogenation forms only one
List I List II
monohaloderivative. Identify the compounds (A) to (E).
(i) Propyne (p) Reduces Fehling’s solution

Q.28 An optically active compound(A),C3H7O2N forms (ii) Ethyl benzoate (q) Forms a precipitate with
a hydrochloride but dissolves in water to give a neutral AgNO3+C2H5OH
solution. On heating with soda lime (A) yields (B) C2H7N. (iii) Acetaldehyde (r) Insoluble in water, but dissolves in
Both (A) react with NaNO2 and HCl the former yielding aqueous NaOH upon heating
a compound (C) C3H6O, which on heating is converted
to (D), C6H8O4 while the latter yields (E), C2H6O. Account (iv) Aniline (s) Dissolves in dil. HCl in the cold
for the above reactions and suggest how (A) may be and is reprecipitated by the addition
synthesized. of alkali

(A) (i) - r, (ii) - q, (iii) - p, (iv) - s


Q.29 An optically inactive acid (A), C5H8O5, on being
(B) (i) - q, (ii) - r, (iii) - p, (iv) - s
heated lost CO2 to give an acid (B), C4H8O3 capable
of being resolved. On action of sulphuric acid, B gave (C) (i) - q, (ii) - r, (iii) - s, (iv) - p
an acid C whose ethyl ester gave (D) on the action (D) (i) - p, (ii) - r, (iii) - q, (iv) - s
of hydrogen and platinum. (D) with conc. NH3 gave
E, C4H9OH which with Br2 and KOH solution gave (F),
C3H9N. F with HNO2 gave G (G) on mild oxidation gave Q.2 Activation of benzene ring by –NH2 in aniline can
H. Both A and H gave the iodoform reaction. Elucidate be reduced by treating with
the reaction mechanism and suggest a synthesis of (C). (A) Dilute HCl (B) Ethyl alcohol
(C) Acetic acid (D) Acetyl chloride
Q.30 A neutral compound (A) C8H9OH on treatment
with NaOBr forms an acid soluble substance C7H9N.
On addition of aqueous NaNO2 to a solution of B in Q.3 Dipolar ion structure for amino acid is
dilute HCl at 0-5ºC, an ionic compound (C) C7H7N2Cl is +
obtained. (C) yields a red dye with alkaline β-napththol (A) H2N CH COOH (B) H3− N CH COO

solution. When treated with potassium cuprocyanide R H2N R
(C) yields a neutral substance (D) C8H7N. ON hydrolysis –
(C) H3+ N CH COO (D) None of these
(D) gives E (C8H6O4). (F) on nitration yields two isomeric
mononitro derivatives (G and H) having molecular H2N R
formula C8H5NO6. Write the reactions involved in
different steps. Q.4 –NH2 group shows acidic nature while reacts with
regent.
(A) Na (B) CS2 (C) Br2+NaOH (D) Water

Q.5 Which of the following does not give ethylamine


on reduction
(A) Methyl cyanide (B) Ethyl nitrile
(C) Nitro ethane (D) Acetamide
2 5 . 4 2 | Amines and Aromatic Compounds Containing Nitrogen

Q.6 Aniline is a weaker base than ethyl amine because (A) Carbylamine reaction
(A) Phenyl gp in aniline is a +R gp (B) Hofmann reaction
(B) Ethyl gp in ethyl amine decreases the electron (C) Gabriel phthalimide synthesis
density on nitrogen atom
(D) Cope reaction
(C) The lone pair of electron on nitrogen atom in aniline
is delocalized over aniline. Q.11 The conjugate acid of HO(CH2)3NH2 is
(D) Aniline is less soluble in water than ethylamine + +
(A) H2O(CH3 )3 NH2 (B) HO(CH2 )3 NH3
Q.7 Diazonium coupling reaction with aniline should be − +
(C) O(CH2 )3 NH2 (D) HO(CH2 )3 NH
carried out in
(A) Weakly basic medium
Q.12 Consider the following compounds:
(B) Weakly acidic medium
1. H2C=CHCH2NH2
(C) Strongly basic medium
2. CH3CH2CH2NH2
(D) Strongly acidic medium
3. HC≡CCH2NH2
The increasing order of basicity is
Q.8 For CH3CHO, CH3NO2, CH3COOH
(A) 3<1<2 (B) 3<2<1
(A) All have same chemical property
(C) 2<1<3 (D) None of these
(B) All have one common chemical behaviour
(C) All are basic
Q.13 Reaction of RCONH2 with a mixture of Br2 and
(D) None of these KOH gives RNH2 as the main product. The intermediate
involved in the reaction is
Q.9 Bromine in CS2 reacts with aniline to give (A) Br O NHBr (B) R–NHBr
NH2 NH2
C
Br
(A) (B) O
Br
(C) R C N (D) R–C=N=O
Br
Br
NH2
Br Br Q.14 Amines are highly soluble in:
(C) (D) Both (A) and (B)
(A) Alcohol (B) Diethyl ether
Br (C) Benzene (D) Water

Q.10 The reaction: Q.15 Which of the following reagents can convert
O O benzene diazonium chloride into benzene?
C C
- +
(A) Water
NH KOH :N:K
C C (B) Acid

O O (C) Hypophosphorous acid


n-BuBr
(D) HCl
O
C
(1)aq. NaOH Q.16 The bromination of aniline produces
N Bu-n n-BuNH2 +
C
(2)H3O+
(A) 2-bromoaniline (B) 4-bromoaniline
O (C) 2,4,6-tribromoaniline (D) 2,6-dibromoaniline
COOH
COOH is called
Chem i str y | 25.43

Q.17 The compound, which on reaction with aqueous Diazonium salts also couple with phenols and aromatic
nitrous acid at low temperature produces an oily amines to form coloured azo dyes. The reactivity of
nitrosoamine is diazonium salts towards coupling reactions is favoured
by presence of electron withdrawing groups; the
(A) Methylamine (B) Ethylamine
reactivity of 2,4,6-trinitrobenzenediazonium chloride is
(C) Diethylamine (D) Triethylamine so high that it even couples with reactive hydrocarbons
such as mesitylene.
Q.18 Carbylamine test is performed in alcoholic KOH
by heating a mixture of Q.20 Consider the following ions:
(A) Chloroform and silver powder
(A) Me2N N N
+

(B) Trihalogenated methane and a primary amine


(C) An alkyl halide and a primary amine
(D) An alkyl cyanide and a primary amine. (B) O2N N N
+

Multiple Correct Choice Type


(C) CH3O N N
+

Q.19 When nitrobenzene is treated with Br2 in


presence of FeBr3, the major product formed is
m-bromonitrobenzene. Statements which are related (D) CH3 N N
+

to obtain m-isomer are:


(A) The electron-density on meta carbon is more than The reactivity of these ions towards azo coupling
that on ortho and para positions reactions under similar conditions is
(B) The intermediate carbonium ion formed after initial (A) I<IV<II<III (B) I<III<IV<II
attack of Br+ at the meta position is least destabilized.
(C) III<I<II<IV (D) III<I<IV<II
(C) Loss of aromaticity, when Br+attacks at the ortho
and para positions, and not at meta position.
Q.21 Which of the following diazonium salts when
(D) Easier loss of H to regain aromaticity from the meta
+ boiled with dil. H2SO4 gives the corresponding phenol
position than from the ortho and para positions. most readily?
OMe
Comprehension Type (A) N N
+
(B) MeO N N
+

Arene diazonium salts are more stable than


alkanediazonium salts due to dispersal of the positive
charge on the benzene ring. Obviously electron (C) Me N+N (D) N N
+

donating groups favour diazotisation by retarding the


decomposition of diazonium salts to phenyl cation.
The high reactively of arenediazonium salts is due to
Q.22 Which of the following arylamines undergoes
the excellent leaving ability of the diazo group as N2
diazotisation most readily?
gas. Therefore, diazonium salts undergo a number
of substitution reactions in which the diazo group is
replaced by a monovalent atom/group such as H (by (A) NO2 NH2 (B) Cl NH2

H3PO2 in presence of Cu+ ions, CH3CH2OH, NaBH4 etc),
OH (by boiling in presence of mineral acids), OCH3 (by
heating with CH3OH) Cl (by CuCl/HCl or Cu/HCl), Br (by (C) CH3 NH2 (D) CH3 NH2
CuBr/HBr or Cu/HBr) I (by KI in presence of Cu+ions), F
(by first converting into N2F4 followed by heating), CN
(by first neutralizing with Na2CO3 and then reacting with
KCN/CuCN), NO2 (by first neutralizing with Na2CO3 and
then treating with NaNO2) phenyl or substituted phenyl
(by treating with benzene or substituted benzene in
presence of NaOH) etc.
2 5 . 4 4 | Amines and Aromatic Compounds Containing Nitrogen

Q.23 The product formed when bromobenzene reacts Reason: Chlorobenzene undergoes nucleophilic
with benzenediazonium chloride in presence of NaOH is substitution by elimination-addition mechanism
while 4-nitrochlorobenzene undergoes nucleophilic
(A) Diphenyl
substitution by addition-elimination mechanism.
(B) p-Bromodiphenyl
(C) p,p’-Dibromodiphenyl Q.30 Assertion: 1° amides react with Br2 + NaOH to
give 1º amines with one carbon atom less than the
(D) p-Bromoazobenzene
parent amide.

Assertion Reasoning Type Reason: The reaction occurs through intermediate


formation of acylnitrene.
Each of the questions given below consists of two
statements, an assertion (A) and reason (R). Select the
Q.31 Assertion: Acetamide reacts with Br2 in presence of
number corresponding to the appropriate alternative
methanoic CH3ONa to form methyl N-methylcarbonate.
as follows:
Reason: Methyl isocyanate is formed as an
(A) If both assertion and reason true and reason is the
intermediate which reacts with methanol to form
correct explanation of assertion, then mark (A)
methyl N-methylcarbamate.
(B) If both assertion and reason are true but reason is
not the correct explanation of assertion, then mark (B)
(C) If assertion is true but reason is false, then mark (C) Previous Years Questions
(D) If both assertion and reason false, then mark (D) Q.1 Benzene diazonium chloride on reaction with
phenol in weakly basic medium gives  (1998)
Q.24 Assertion: Benzyl amine is more basic than
(A) Diphenyl ether (B) p-hydroxy azobenzene
aniline.
(C) Chlorobenzene (D) Benzene
Reason: Positive inductive effect of phenyl group
creates high electron density around N atom.
Q.2 (2007)
Q.25 Assertion: White precipitate of silver chloride (CH3)2NH
gets dissolved in NH4OH solution. F NO2 DMF,∆
(A)
Reason: NH3 reacts with AgCl to form a solution (i)Fe/HCl
complex with formula [Ag(NH3)2]Cl. (ii) NaNO2/HCl/0ºC
(iii) H2/Ni
(B) is
Q.26 Assertion: o-nitrophenol is more acidic than
p-nitrophenol. CH3
(A) H2N N (B) H2N NH2
Reason: Nitro group has +M and –I effect. CH3

Q.27 Assertion: 3°amine is proved to be less basic in


(C) O N CH3 (D) O2N NH2
aq. solution 2 N

H2N
Reason: Conjugate acid of 3°amine is poorly solvated
in aq. solution.
Q.3 Amongst the compounds given, the one that would
form a brilliant coloured dye on treatment with NaNO2
Q.28 Assertion: In order to convert R–Cl to pure R–NH, in dil. HCl followed by addition to an alkaline solution
Gabriel-phthalimide synthesis can be used. of β -naphthol is (2011)
Reason: With proper choice for alkyl halides, phthalimide N(CH3)2 NHCH3
synthesis can be used to prepare 1°, 2° and 3° amines. (A) (B)

Q.29 Assertion: 4-Nitrochlorobenzene undergoes NH2


nucleophilic substitution more readily than CH2NH2
chlorobenzene. (C) H3C (D)

Chem i str y | 25.45

Q.4 Match the Columns Q.11 In the following reaction the structure of the
major product ‘X’ is:  (2007)
Match the compounds in column I with their
characteristic test(s)/reaction(s) given in column II.
 (2010) O

Column I Column II N conc.HNO3


X
H conc.H2SO4
(p) Sodium fusion extract
⊕ O

of the compound gives
(A) H2N — NH3 Cl
Prussian blue colour with
O
FeSO4
(A) NO2

(q) Gives positive FeCl3 N
NH2I test H
(B) HO
COOH
O2N
 (r) Gives white precipitate O
(C) HO NH3Cl with AgNO3
(B)
N
H
 (s) Reacts with
(D) O2N NHNH3Br aldehydes to form
NO2 the corresponding O
hydrazone derivative.
(C)
N
H
Q.5 In an acidic medium, ..............behaves as the NO2
strongest base. (nitrobenzene, aniline, phenol) (1981)
O
Q.6 The high melting point and insolubility in organic (D)
solvents of sulphanilic acid are due to its ................ O2N N
structure.  (1994) H

Q.7 State the equation for the preparation of following


Q.12 Among the following, the least stable resonance
compound. Chlorobenzene from aniline (in two steps)
structure is (2007)
 (1982)
+ O
+ + O
Q.8 State the conditions under which the following (A) -
+
N
N
preparation carried out. (A) -
O
“Aniline from benzene” (1983) O
-
-
- O
- + O
+
Q.9 Complete the following with appropriate structures: (B) + N
N
(B) +
 (1986) O
O
-
Base -
NH2 + COCl ? O
-
- + O
+
N
(C) +
(C) + N
Q.10 Write the structure of the major organic product O
O
-
expected from the following reaction. (1992) -
-
-O
CH3 + O
N + HNO2 (D) +
N
(D) +
+ N
CH3
O
O
-
-
2 5 . 4 6 | Amines and Aromatic Compounds Containing Nitrogen

Q.13 Statement-I: Aniline on reaction with NaNO2/HCl (iii) NO2 (r) Scheme III
at 0oC followed by coupling with β- naphthol gives a
(i) red hot iron, 873(ii) fu mingHNO3 ,H2SO 4 , heat
dark blue coloured precipitate. (ii) H S.NH (iv) NaNO ,H SO (v)hydrolysis
2 3 2 2 4
?  → C6H5NO3
Statement-II: The colour of the compound formed in C6H5NO3
the reaction of aniline with NaNO2/HCl at 0oC followed
by coupling with β-naphthol is due to the extended (iv) NO2 (s) Scheme IV
conjugation. (2008) (i) conc . H2SO 4 , 60o C
(ii) conc . HNO3 , conc. H2SO 4 (iii)dil. H2SO 4 . heat
(A) Statement-I-I is True, statement-II is True; ? 
→ C6H5NO 4
statement-II is correct explanation for statement-I CH3 C6H5NO4

(B) Statement-I is True, statement-II is True;


statement-IIis NOT a correct explanation for statement-I Code:
(C) Statement-I is True, statement-II is False (i) (ii) (iii) (iv)
(D) Statement-I is False, statement-II is True (A) p s q r
(B) r p s q
Q.14 Match the compounds in column I with their
(C) r s q p
characteristic test(s)/ reaction(s) given in column II.
Indicate your answer by darkening the appropriate (D) s p r q
bubbles of the 4 × 4 matrix gives in the ORS  (2008)

Column I Column II Q.16 The major product of the reaction is  (2015)

+ −
(p) sodium fusion extract of
(A) H2N − NH3 Cl the compound gives Prussian H3C CO2H
NaNO2, aqueous HCl
blue colour with FeSO4 o
CH3 0C
NH2
 (q) gives positive FeCl3 test
NH2I
(B) HO
COOH (A) H3C NH2 (B) H3C CO2H

 (r) gives white precipitate CH3 OH CH3 OH


(C) HO NH3Cl with AgNO3 (C) H 3C CO2H (D) H3C NH2

CH3 OH CH3 OH
(D) (s) reacts with aldehydes
 to form the corresponding Q.17 In the following reactions, the product S is
O2N NHNH3Br
hydrazone derivative  (2015)
NO2 H3C
i. O3 NH3
R S
ii. ZnH2O
Q.15 Match the four starting materials (P, Q, R, S) given
in list I with the corresponding reaction schemes (I, II,
III, IV) provided in list II and select the correct answer
H3 C H3C N
using the code given below the lists.  (2014) N
(A) (B)
List I List II
(i) H H (p) Scheme I
(i) KMnO 4 , HO− , heat (ii), H2O
(ii) SOCl2 (iv) NH3 N
→ CC77H66N
?  N22OO3 3 N
(C) (D)
(ii) NO2 (q) Scheme II H3C H3C

(i) Sn/HCl (ii) CH3COCl (ii) conc H2SO 4


(ii) NHO3 (v) dil.H2 SO 4 , heat (vi) HO−
? → C C
6HHNNOO
66 62 2 22
Chem i str y | 25.47

Q.18 In the following reactions, the major product W is 

OH  (2015)
NH2
NaNO2, HCl , NaOH
o V W
0C

OH
(A) (B)
N=N OH N=N

HO

OH
(C)
(D)
N=N N=N

Q.19 The product (s) of the following reaction sequence is (are) (2016)

NH2 (i) Acetic anhydride/pyridine


(ii) KBrO3/HBr

(iii) H3O+, heat


(iv) NaNO2/HCl, 273-278 K
(v) Cu/HBr

Br Br Br Br
Br Br Br Br
(A) (B) (C) (D)
Br

Br Br
2 5 . 4 8 | Amines and Aromatic Compounds Containing Nitrogen

Paragraph: Treatment of compound O with KMnO4/H+ gave P, which on heating with ammonia gave Q. The
compound Q on treatment with Br2/NaOH produced R. On strong heating, Q gave S, which on further treatment
with ethyl 2-bromopropanoate in the presence of KOH followed by acidification, gave a compound T

Q.20 The compound R is  (2016)

O O O
NH2 Br NHBr
(A) (B) (C) (D) NBr
NH2 Br NHBr

O O O

Q.21 The compound T is  (2016)


(A) Glycine (B) Alanine (C) Valine (D) Serine

PlancEssential Questions
JEE Main/Boards JEE Advanced/Boards

Exercise 1 Exercise 1
Q.1 (iii) Q.3 Q.12 (iii) Q.2 Q.6 (4) Q.7 (4)

Q.17 (ii, vi) Q.19 (iv) Q.22 (ii) Q.14 Q.23 Q.29

Q.27

Exercise 2 Exercise 2
Q.10 Q.14 Q.19 Q.8 Q.19 Q.20
Q.22 Q.28

Previous Years’ Questions Previous Years’ Questions


Q.1 Q.5 Q.10 Q.2 Q.4
Chem i str y | 25.49

Answer Key

JEE Main/Boards
Exercise 2
Single Correct Choice Type

Q.1 A Q.2 C Q.3 D Q.4 B Q.5 D Q.6 C


Q.7 D Q.8 A Q.9 D Q.10 B Q.11 D Q.12 A
Q.13 D Q.14 B Q.15 D Q.16 B Q.17 D Q.18 D
Q.19 B Q.20 A Q.21 B Q.22 A

Previous Years' Questions


Q.1 C Q.2 A Q.3 C Q.4 B Q.5 C Q.6 C
Q.7 A Q.8 A Q.9 A Q.10 A Q.11 A Q.12 B
Q.13 B Q.14 C Q.15 B Q.16 D Q.17 A Q.18 C

JEE Advanced/Boards
Exercise 2
Single Correct Choice Type

Q.1 B Q.2 D Q.3 C Q.4 C Q.5 B Q.6 C


Q.7 B Q.8 B Q.9 C Q.10 C Q.11 B Q.12 A
Q.13 A Q.14 D Q.15 C Q.16 C Q.17 C Q.18 B

Multiple Correct Choice Type


Q.19 A, B

Comprehension Type
Q.20 B Q.21 A Q.22 C Q.23 B

Assertion Reasoning Type


Q.24 C Q.25 A Q.26 D Q.27 A Q.28 C Q.29 B
Q.30 A Q.31 A

Previous Years Questions


Q.1 B Q.2 A Q.3 C Q.4 A → p; B → q; C → r; D → s Q.11 B
Q.12 A Q.13 D Q.14 A → r, s; B → p, q; C → p, q, r; D → p, s Q.15 C
Q.16 C Q.17 A Q.18 A Q.19 B Q.20 A Q.21 B
2 5 . 5 0 | Amines and Aromatic Compounds Containing Nitrogen

Solutions

JEE Mains/Boards Sol 8: (i) Sandmeyer Reaction is used to synthesize


aryl-halides from aryl amines.
Exercise 1
CH3 CH3 CH3
NH2 NaNO2 N+2Cl- CuCl, HCl Cl
Eg. HCl 60C
NH > 0.5C
Sol 1: (i) NH > NH2 > NH2
CH3 (ii)

(ii) C6H5NH2 < C6H5NHCH3 < C2H5NH2 < (C2H5)2NH


⊕ ⊖
(iii) (a) p-nitro aniline < Aniline < p-toluidine N2 Cl ∶OH

(b) C6H5NH2 < C6H5NHCH3 < C6H5CH2NH2 +

Sol 2: 1º and 2º amines react with acetyl chloride to



form acetyl derivatives. O—H N⚌N OH

R–NH2 + CH3COCl → RNHCOCH3 + HCl


(CH3)2–NH + CH3COCl → (CH3)2N–COCH3 + HCl ⊖

Tertiary amine do not undergo this reaction.


Sol 9: (i) Refer to Sol.8 (ii)
(ii) The reaction involves the conversion of an amide
Sol 3: O
O
into one carbon loss, by action of hypoactive NaOH
Br2
C
H2 O
R NH2 solution +Br2.
R NH2 NaOH R N -CO2
O
Reaction of a primary amide to form primary amine Example: CH3—C—NH2 Br2/KOH
CH3NH2 + CO
with one fewer carbon atom.
(iii) Acetylation - 1º or 2ºC amines react will acetyl
Sol 4: +ve charge or secondary carbon is not stable chloride acetic anhydride to from acetyl derivatives.
Ex: CH3NH2 + CH3COCl → CH3NCOCH3 + HCl
Sol 5: Aliphatic amines
Primary: RNH2 + HNO2 → ROH + H2O + N2 Sol 10: (i) Aniline is a weaker base than methyl
compound
R R
Secondary: NH+HNO2 N-N=O In case of aniline, the lone pair is delocalised on the
R R +H2O ring and hence it is less basic.
+ –
Tertiary:R3N + HNO2 → R3NH + NO2 Cl CN
(ii) + NaCN
Aromatic Amines forms diazonium ions.
Cl– is a stronger base than CN– hence the reaction can
Sol 6: (i) Refer Sol.3 take place, + there is not sufficient + ve on the for
(ii) Refer Gabriel Pthalimide synthesis nucleophilic substitute to take place.

Sol 11: Refer to sol. 2


Sol 7: (a) Lone pair delocalised in aryl amines.
(b) CH3NH2 → CH3OH → CH3I → CH3CN
Sol 12: (i) Aniline is a weaker base as compared to
→ CH3CH2NH2 methyl amine as lone pair of NH2 is delocalised on the
ring. Hence pkB aniline > pkB methyl amine.
Chem i str y | 25.51

(ii) MeNH2 is a good base in water,


O it reacts will FeClO
3
to O
form Fe(OH)3 + MeNH+4 Cl− OH
PCl5
Cl
KCN
CN
LiAlH4 NH2
(iii) Due to NH2 group on aniline, there is extra e– density
NaNO2+ H2SO4
on benzene ring. Hence, nucleophilic substitution does
not take place. O 
Kr2Cr2O7/H2SO4 N2Cl
N2 +,Cl- NO2
OH
Sol 13: (i) HNO2
(vi) CH3NH2
HCl (i)NaNO2/HCl NH2
low FeCl3 (ii)KCN
(iii)LiAlH4
NH2 N=N+Cl-
NH
(ii) NaNO2 (vii) CH3NO2 (i)NH3
(ii)NH2 CH3
HCl
(viii) (i) NH3/H2SO4
O COOH (ii) NaNO2/HCl CH3 COOH
Br2 /KOH (iii) K2Cr2O7/H2SO4
(iii) CH3—C—NH2 → CH3—NH2 + CO

Sol 18: (i) Any reactions which involves two or more


equivalent of reactions for proceeding reaction are
Sol 14: Factual known as coupling reaction

NH2 N2+Cl Exp. Wurtz,Pinacol-pinacolone etc.


(ii) Addition of ammonia → Ammonolysis
NaNO2
Sol 15: HCl O
(iii) Addition of acetyl group CH3—C— acetylation
Sol 16: (i) In aromatic amines, the ⊕ charge is delocalised
on the less electron -ve ring C- due to good resonance. (iv) Gabriel Synthesis:
(ii) Due to the smaller size of 1º amine it easer to attack
and hence. phthalimide synthesis is preferred for any O
N
amine. R NH
N2H4
R Cl O R NH2 +
O
NH
Sol 17:
NK
NH3 /H2SO 4
(i) —COOH  → CH
 3
—NH2 O
Test for 1 amines
(i)H2O/H
+ +
O
(ii) N Br2/4KOH O NH2
R N NH
N2H4
R Cl
(i) CHCl R NH2 +
(iii) CH3OH  O 3 /HCl → CH
O
—COOH
(ii) 3KOH 3 NH
NK
(iv) NH2
(i) NaHO2/HCl
O (ii) H O OH
2 Test for 1 amines
K2Cr2O7/H2SO4

O
HN3/H2SO4
CH3 NH2 + CO2 OH

(v) O O O
OH
PCl5 KCN
CN
LiAlH4 NH2
Cl
NaNO2+ H2SO4

O 
Kr2Cr2O7/H2SO4 N2Cl
2 5 . 5 2 | Amines and Aromatic Compounds Containing Nitrogen

NO2 COOH (ix) NH2


(i) NaNO2/HCl
OH
(ii) KCN
Sol 19: (i) (i) NaNH2/KOH
(iii) LiAlH
(ii) CHCl3/HCl, 
(iv) NaNO2/H2O

(ii)
OSO3H
OH OSO3H
Sol 20: Refer Text for Curtius mechanism.
H2O2/Fe
2+
Oleum Br2/H2O
Br
OH Sol 21: (i) ph—N ≡ C
H2SO4/H2O
(ii)
Br

(iii) (iii) Acid-base reaction ph- NH3 + H2SO −4
COOH COCl CONH2 NH2 (iv) phOEt
SOCl2 NH3 Br2/
NaOH
NH2 NH2
Benzoyl Benzamide Aniline Br
Benzoic acid
chloride (v) +
(iv) (Major) (Minor)
Br
NH2 NH2 F
Br Br (i) (CH CO) O Br Br (vi) Acetophenone
Br Br
Br2/H2O 3 2 AgF
excess (ii) H3PO2 (vii) Nitrobenzene
Br Br Br
Sol 22: (i) Electronegativity of ‘O’ > ‘N’
F
Br Br (ii) More surface area ⇒ more interaction b/w molecular
Br Br AgF
⇒ higher B.P.
(iii) In aromatic amines,they have lone pair of N in
Br
Br conjugation with ring ⇒ less basic
CH2 Cl
NH2
(v) (i) KCN
NH2
(ii) LiAlH4 Sol 23: (i) (i) NaNO2/HCl
(ii) LiAlH4
Cl

Cl (ii) CH3NH2 


(i) NaNO2 /HCl
→ CH3CN
(ii) KCN
(vi)
+
(i) H2O/H
(i) HNO3/H2SO4 (iii)
(ii) LiAlH4
N (ii) Br2/KOH NH2
NH2
Schmidt reaction
(vii)
NH2 OH
NH2 NH2
NH2
(i)H2SO4 (2eq’s)
SO3H SO3H +
(iv) (i) NaNO2/HCl
H /H2O
(ii)Br2/H2O (ii) KOH
Br Br
Br Br
NO2 NH2
CONH2 CH3
Br Br
(viii) . (v) (i)NaNO2/HCl
(i) H2/Ni (ii)NH3
(ii) NaNO2/HCl (iii)Br2/H2O (excess)
(iii)LiAlH4 Br
Chem i str y | 25.53

Sol 24: Intramolecular acid-base reactions of molecule Sol 6: (C) End product has a hydrophobic and a
hydrophilic part.
(like some amine + carboxylic acid in same molecule)

Sol 7: (D) s > t > p > NH3


NH2 NH2
NO2 NO2
Sol 25:
(i)(CH3CO)2O Sol 8: (A)
(ii)HNO3/H2SO4
(iii)H+/H2O
NO2 localised lone pair

:
N
Sol 26: because carbocation intermediate in Friedel
craft alkylation reach with l.p. of N in aniline
Sol 9: (D) CH3NO2 
Cl2
→ CH2ClNO2 

Cl2


NaOH NaOH

COOH CHCl2NO2  → CCl3NO2


Cl2
(i) NH3 NaOH
(ii) LiAlH4
Br
Sol 27: (iii) Br2/H2O CH3NH2 is not formed
(iv) K2Cr2O/H2SO4
NO2
Sol 10: (B) Br2/NaOH
Br
R—CH2CONH2 2 → R—CH2–CH
Sol 28: N due to flipping it is considered as NaOH

Sol 11: (D)


planar molecule.
CH3
NaNO2 N N=O
NHCH3
Exercise 2
HCl

Single Correct Choice Type H2N


SO3H Sol 12: (A) NH
C
H2N
Sol 1: (A) (i) → + H2SO4 + H2 O

NH2 NH2 Sol 13: (D) All of these


K2Cr2O7 is a strong oxidising agent.
H
H H
N Sol 14: (B) O
Cl Cl
C
Sol 2: (C) Not acceptable NH2 N
H

O H

Nitrogen can’t form 5 bonds + C


Cl Cl
Sol 3: (D) Order of basic strength
4 > 3 > 2 > 1 Basic strength ∝ +I effect O Cl
C
Sol 4: (B) 1 > 2 > 3
N=C=O NH

Sol 5: (D) Primary amines gives carbylamine test.


2 5 . 5 4 | Amines and Aromatic Compounds Containing Nitrogen

Sol 15: (D) CH3—CH2—NH2 + KMnO4 → CH3CHO Sol 3: (C) Hofmann degradation of amide
R–CONH2 + Br2 + 4KOH →
Sol 16: (B) Baker-Mulliken’s Test is a test for R – NH2 + 2KBr + K2CO3 + 2H2O
nitrocompounds.
(pr imary amine)

Sol 17: (D) R2


R1 R3 Sol 4: (B) (CH3)2NH>CH3NH2>(CH3)3N
N
N Kb=5.4 ×10–4, 4.5×10–4, 0.6×10–4
R1 R3
R2
flips to Sol 5: (C) (CH3)2NCOCH3 + HCl/H2O →

Due to flipping, tertiary amine is a racemic mixture. (CH3)2NH + CH3COOH

O Sol 6: (C) Secondary>tertiary>primary


Sol 18: (D) CH3 shows H-bonding,
N
O - Sol 7: (A) Because OH– is nucleophile.
α -halogenation as well as tautomerism.
Sol 8: (A) It is true that benzene diazonium chloride
Sol 19: (B) Nitrobenzene on reduction with Sn/HCl does not respond Lassaigne test of nitrogen because
gives aniline. With Zn it gives phenyl hydroxylamine. benzene diazonium chloride loses N2 on slight heat
With methanolic NaOMe nitrobenzene gives and thus it can’t react with sodium metal.
Azoxybenzene. On reaction with Zn and strong base it
gives Hydrazobenzene. Sol 9: (A) Amines are basic due to the presence of a
lone pair of electrons on nitrogen atom. The lone pair
Sol 20: (A) RCN < RCH = NR < RH2N can be easily donated.
tertiary secondary primary

Sol 10: (A) In an isocyanide, first an electrophile and


Sol 21: (B)
then a nucleophile add at the carbon to form a species
CH3 which usually undergoes further transformations.
CH3 CH3 + −
NH2 RN ≡ C+ H2O →=
RN CHOH → RNHCHO
Alkyl Formamide

NH2 , NH2 ,
Sol 11: (A) self-explanatory
Sol 22: (A)
Sol 12: (B) Due to strong deactivation of benzene ring
O OH NH2 NH2 by –NO2 group, nitrobenzene is often used as a solvent
LiAlH4
+ NH2OH in Friedel–Craft’s reaction.

Sol 13: (B) -NO2 is electron withdrawing which will


destabilize σ - complex.
Previous Years’ Questions
Sol 14: (C)
Sol 1: (C) Self- explanatory
+ -
NH2 N2 Cl F

Sol 2: (A) CH3CONH2 +Br2+4KOH 


–2H O
→ NaNO2 HBF4
2 + N2 + Bf3 + HCl
(Acetamide) HCl, 278 K

CH3NH2 +2KBr+2K2CO3 (A) (B)


Benzene diazonium Fluorobenzene
(Methyl amine) chloride
Chem i str y | 25.55

Sol 15: (B) Molisch’s Test: when a drop or two of JEE Advanced /Boards
alcoholic solution of a–naphthol is added to sugar
solution and then conc. H2SO4 is added along the sides
of test tube, formation of violet ring takes place at the Exercise 1
junction of two liquids.
Sol 1: (i) Carboxylic acid, amine, ester
CHCl3 /KOH
Sol 16: (D) R

− CH2 − NH2 
C2H5OH
→R − CH2 − NC NH3 CH2C6H5
(ii) OOC CH2 CH C NH CH COOCH3
Sol 17: (A) Among C6H5NH2, CH3NH2, (CH3)2NH,(CH3)3N.
O
C6H5NH2is least basic due to resonance.
(iii) Aspartic acid and phenylalanine
+ +
NH2 NH2 NH2 O
NH2
- OH
NH2
HOOC CH2 CH COOH
-
+ (iv) Phenylalanine is more hydrophobic because of the
NH2 NH2
benzyl side chain.
-

Sol 2:

Out of (CH3)3N, CH3NH2, (CH3)2NH. (CH3)2NH is most CHO COOH N(CH3)2


basic due to +I effect and hydrogen bonding with H2O. KMnO4 Sada-lime

CH3 +I effect
N(CH3)2
:

N(CH3)2
N

H3C
+I effect
H Sol 3:
:

O HNO2 KMnO4

Hydrogen H H NH2 OH O
bonding (A) (B) (C)
Other isomers are primary amines.

Sol 18: (C)


+ - NO2
NH2 N2Cl CN

NaNO2,/HCl Sol 4:
CuCN/KCN
+ N2
o
0-5 C 
NO2
CH3 CH3 CH3
(D) (E)
Sol 5: (i) + HCl

NH2 NH3Cl

••
(ii) CH3—CH2— NH2 Ethylamine
2 5 . 5 6 | Amines and Aromatic Compounds Containing Nitrogen

O Sol 9: (i) Steric hinderance

CH3 C NH2 Acetamide (ii) Isocyanides are stable in strong basic conditions, but
they are sensitive of acids.
Ethylamine is basic due of the presence of localised O O
lone pair. (iii) CH3 N CH3 N +H


Acetamide is acidic due to the presence of acidic O O


hydrogen.
O
Stable
N CH3 N CH3 CH3 N
(iii) H CH3 O
CH3 CH3
(iv) Aniline forms a salt with AlCl3. Due to this nitrogen
Dimethylamine Trimethylamine atom of aniline acquires a positive charge and hence
acts as a strong deactivating group and doesn’t allow
More steric hinderance in Trimethylamine
the reaction to take place.
SO3- (v) The basicity of nitrogen in trimethylamine is greater.

(iv) It forms zwitter ion


Sol 10:

NH3
 (i) C=C6H5COCl D=C6H5CONH2
E=C6H5CHNH2
Sol 6: (i) Amine group in glycine is more basic. (ii) F=C6H5CN; G=C6H5COOH
(ii) More equivalent resonance structures (iii) H=EtNHBr
(iii) It stabilises the complex. N(CH3)2
(iv) Because of side products
(iv) I=

Sol 7: (i)
NO
Me NO2
CH3 NH n-propylamine forms H-bond.
N (v) J =O2N N N Me
Me Me CH3

(vi) K=C6H5SO3H ; L= C6H5SO3Na


(ii) Steric hinderance in trimethylamine
(iii) Due of formation of a soluble complex salt M= C6H5OH
OH
AgCl + 2CH3NH2 → [Ag(CH3NH2)2]+Cl–

(vii) I = NaOH N =
Sol 8: (i) Ethylamine in aqueous solution reacts with
ferric chloride to precipitate hydrated ferric acid.
SO3H OH
(ii) No N—H bond in tertiary amine.
N (viii) O = P=
SO3H OH

(iii) Steric hinderance of 2 Me groups OEt

present on benzene ring.


(iv) In (I) lone pair is localized, Inductive effect of ‘O’ in (III) (ix) Q = Ph–O–Et R=

In IV the lone pair is delocalised CHO

I > II > III > IV


S = EtO CH=N–NH–Ph
Chem i str y | 25.57

Sol 15:
(x) T = CH3–C–NH- + para isomer
OCH3
O Br
Cl (A) Cl—C—CH2CH2Br

+ N2 CH3
(xi) U = Cu Powder V =
OCH3
⊕ ⊖ (B) H3C—C—CH2CH2NH2
NH2 NH2Cl
Sol 11: (A) (B) CH3
Et Et
OCH3
CN COOH
(C) C—C—CH2CH2NH–CH3
(C) (D)
Et E CH3
COOH COOH OCH3

(E) (F) (D) H3C—C—CH2CH2OH


C—CH3 C—CH3

Cl Cl O CH3
COOH OCH3 CH3
COOH
(G) (H) (E) H3C—C—CH2CH2N—CH3
C—OMe
COOH
O CH3
C
O Sol 16 (i)
(I)
C
CH2CH3
CH2CH3
O O+CH3CH2NH2
O+CH3CH2NH2
N
N
CH2CH3
Tautomerise N CH2CH3
Sol 12: C2H5NH2CH3—CH2—NH2 Tautomerise N
H
H
(ii) CH2CH3
Sol 13: O+(CH3CH2)2NH2 N CH CH3
CH2CH32
 O+(CH3CH2)2NH2 N
CH3 NH2 Cl CH3 NH CH2CH3
H H
NaOH H H
Tautomerise N(CH2CH3)2

Tautomerise N(CH2CH3)2

Sol 14:
Sol 17:
CH2OH CH2Cl CH2 CN CH2 CH2 NH
NH2 NH2
PCl5 KCN Na H
CH2 CH2 CH2 CH2
EtOH
CH2OH CH2Cl CH2 CN CH2 CH2 NH2
Cyclohexylamine Aniline


Lone pair of nitrogen in aniline is delocalized due to


NH resonance.
OH
COOH NO2 NO2
OHHNO3
B = and Aromatic excess
2 5 . 5 8 | Amines Compounds Containing Nitrogen
NO2
Sol 18: Use excess OH
of ammonia to reduce the chances
COOH Cl
of reaction of NO
1º amine with
NO2 alkyl halide
NO2 to formNO2º
2 and NO2 NO2 NO2 NO2
3º amines. CO2 PCl5

Sol 19: Amino groupNObeing


2 activating group
NO2 activates NO2 NO2
bromination of aniline and forms tribromoaniline.
NH OH.HCl
Sol 24: C8H8 O 
2 →B + C
Sol 20: Although inductive effect of alkyl groups is Ketone/
greater in trimethylamine but the alkyl groups crowds → D
B 
acid
the nitrogen atom and reduces its basicity.
acid
C 
→ E
Sol 21: D + alc. KOH → F
CH3 CH3 F + CH3COCl → D
hydrolysis
N CHO NH + HCOOH E alkali → acid/solid C7H6O3 (G)
CH3 CH3 C8H8O → Double bond equivalent
(A) (B) (C) 1
DBE = ((16+2)–8) = 5 → benzene ring and a π
2
Sol 22: NH2 bond outside.
∴ NH2OH/HCl → 2 product (G.I.)
∴ It is a ketone.
NH2 Alkene
Optically active amine ozonolysis
O
Ph
HCHO + CH3CH2CH2CHO
 (A) CH3 NH2OH N
No, there is no structural isomer possible. HCl
CH3 OH
(2 G.I.)
Sol 23: cis and trans

OH O
Ph
COOH COOH Ph
N acid NH CH3
NH2 NaNO2 OH
HCl (D)
(B) (Beckman Rearrangement)

Ph
The compound gives +ve less for NH2 and COOH N acid Ph NH
CH3
OH
O
COOH
(E)
NH2
 A is O
Ph O
OH NH CH3 alc. KOH
Ph N H2 CH3 C OH
COOH NO2 NO2
OH HNO3 O
B= excess CH3 C Cl
NO2
OH COOH Cl O
NO NO2 NO2 NO2 NO2 NO2 NO2 NO2 Ph NH C CH3
(Hydrolysis)
CO2 Ph PClNH
5 alc. KOH
Ph COOH + CH3 NH2
CH3
(White solid)
NO2 NO2 NO2 O NO2
(G)
(E)
Chem i str y | 25.59

Sol 25:
[O y ], ∆
F 
→ G
(A)
Sn/HCl NaNO2/HCl
C7H5NO2Cl2 (B) (C)
{does not form dye +G
with β -napthol} given CAN test AlCl3
B = C7H5NH2Cl2 does not give compound B-does not
form a dye with β -naphthol
nitration (D) (O)
(F) (E) –COOH
(forms single mono- (acid) ∴ A is (C) is a primary alcohol.
nitro derivative) eq.wt = 191
as de-carboxylation gives single derivatives
∴ Cl should be symmetrically oppositely placed.
Cl Cl
Cl Cl Cl C
CH2NO2 CH2NH2 CH2O
CH2NO2 CH2-NH2
(A) (B)
A=

Cl Cl Cl Cl Cl C

Cl
Cl Cl Cl Cl Cl
CH2NO2 CH2-NH2 CH2O COOH
CH2OH A = COOH
(C) (D)
Cl Cl Cl Cl
Cl Cl Cl Cl
NO2
Cl Cl
NO2
(E) (F) Cl Cl

Cl Cl
Sol 27:
NH2 OH
Sol 26: ∴ NH+4
as gas is NH3
(A) (B)
−NH2 + OH
M.W. 135 → B mole IRH
alc. CH3 CH3
→ R—CH2COOH → Mass of R =
A + NaOH 
NC CN
R—CH2—NH2 
HNO2
→ RCOOH (B)
Br /KOH
A →
2 C primary amine R–CH2–NH2 (C) (D)
cold
R–CH2–CONH2 (C)  → (D) Primary amine
 red
→
HNO 2 CH3 CH3
cold CAN

dil HCl COOH


E 
→ F (mol. wt 136)

(E)

COOH
2 5 . 6 0 | Amines and Aromatic Compounds Containing Nitrogen

Sol 28: Degree of unsaturation of A = 2 since A CH2OH


forms hydrochoride, it contain both a basic and acidic
functional group. It’s most likely to be an amino acid. A = H3C—C—CO2H
On decarboxylation, it forms an amine B, Degree of
unsaturated of B = 0 CO2H
∴ B is saturated amine.
CH2OH
NaNO2
∴B 
HCl
→ give C2H5OH (E)
B = H3C—C—CO2H
∴ B is CH3CH2NH2
H
NaNO2
 → C (a hydroxyl acid) Which yeilds cyclic OH
HCl

diester on heating C = H3C—CH—CO2H

D = (CH3)2CHCO2C2H5
∴ Reactions
E = (CH3)2 CHCONH2
NaOH
H3C—CH—COOH 
CaO

→ : CH3 – CH2 – NH2 F = (CH3)2 CHNH2

NH2 G = (CH3)2 CHOH

CH3 – CH2 – NH2  H = CH3COCH3


→ CH3CH2OH
NaNO2
HCl

Sol 30:
NaNO2
H3C—CH—COOH  → H3C—CH—COOH
HCl CONH2 NH2
NH2 (A) (B)
OH
(c) CH3 CH3
O +
N2Cl– CN
C (C) (D)
O CH-CH3
H3C2 CH COOH 
CH3 CH3
-H2O
CH3-HC O
OH C
CO2H COOH
O (E) (F)
P/Br
H3C2—CH—COOH CH3 COOH
CH3–CH2–COOH →
2

Br CO2H CO2H
NH3 (G) (H)
 → H3C2—CH—COOH
CO2H O2N CO2H
NH2

H SO
Sol 29: (A) C5H8O5 → (B) C4H8O3 
2 4→

Exercise 2
H conc. NH3
C 
alkyl ester
2 → D 
→  → E (C4H9OH)
Single Correct Choice Type
Br
C3H9N ←2  (C H OH)
KOH 4 9 Sol 1: (B) Propyne- Forms a precipitate with AgNO3 in
degree of unsaturation = 2 × 5 + 2 – 8 ethanol.

=4–2=2 Ethyl Benzoate -Insoluble in water, but dissolves in


aqueous NaOH upon heating
Chem i str y | 25.61

Acetaldehyde- Reduces Fehling’s solution Br


Aniline- Dissolves in dilute HCl in the cold and is
reprecipitated by the addition of alkali. Sol 9: (C) Br2
Cs2
Br Br
Sol 2: (D) Acetic acid NH2 NH2

Sol 3: (C) H3N+ —CH—COO–


Sol 10: (C) Gabriel-phthalimide synthesis.
R
Sol 4: (C) Sol 11: (B)
H
 
H NH+ HBr+Br HO CH2 CH2 CH2 NH2 HO (CH2)3 NH3
N Br Br
H
Sol 12: (A) 2 > 1 > 3
Sol 5: (B) Ethylnitrile (CH3CN)
Consider inductive effect of primary, secondary and
tertiary law.
Sol 6: (C) The lone pair of electron on nitrogen atom in
aniline is delocalized over aniline. O O
Br-Br
•• Sol 13: (A) R C NH2
KOH
R C NH
Sol 7: (B) R — NH2 behave as a base in weakly acidic
medium Br

Sol 14: (D) Self-explanatory


NH2
+2HCl
+NaNO2
. Sol 15: (C)
-2H2O
-NaCl
 Cl
OH NN

OHH PO
3 2

Cl + N2
 N
N N
HCl N Sol 16: (C)

OH
Br
OH
N
N Br
2 →
N NH2

H O Br Br
HCl 2

NH2
O
O O Sol 17: (C) Secondary amine
CH3 N
Sol 8: (B) CH3 C H CH3 C OH O-
Sol 18: (B) Trihalogenated amine and a primary amine
All three shows acidic behaviour.
2 5 . 6 2 | Amines and Aromatic Compounds Containing Nitrogen

Multiple Correct Choice Type R


Sol 27: (A) Large size less solvate or
Sol 19: (A, B) The electron density on meta carbon N
is more than that on ortho and para positions. The
intermediate carbonium ion formed after initial attack R R
of Br+ at the meta position is least destabilized due to
hence less basic.
no mesomeric effect.

Sol 28: (C) 3º amine cannot be synthesised using


Comprehension Type
Gabriel phthalimide
Sol 20: (B) NO2 → –M effect
Sol 29: (B) Both statements are true but
CH3 → +Z effect
4-nitrochlorobenzene goes nucleoliphilic substitutions
OCH3 → mid +M effect more readily than benzene due to the M-effect of
nitro group 2 +ve charge is created on C attaches to
NMe2 → high +M effect
Cl Molecule

Sol 21: (A) –I effect of –OMe


O
Br2/NaOH
Sol 22: (C) +M effect of –OMe at para position Sol 30: (A) R—C—NH2 R—NH2

Sol 23: (B) The product formed when bromobenzene Sol 31: (A) Self explanatory (refer to theory of reaction)
reacts with benzenediazonium chloride in presence of
NaOH is p-Bromodiphenyl

Assertion Reasoning Type Previous Years’ Questions

Sol 1: (B) C6H5 N+ Cl– + C6H5OH →


2
NH2
N=N OH
Sol 24: (C)
p-hydroxy azobenzene
N
Sol 2: (A)

Sol 25: (A) False lone pair is delocalised of Ph-NH2


whereas its localised for aniline . F NO2
(CH3)2NH CH3
DMF CH3 N NO2
Self Explanatory
(i)Fe/HCl
NO2 NO2 (ii)NaNO2 CH3
(iii)H2/Ni N NH2
OH CH3

Sol 3: (C) As we know, benzenediazonium salt


Sol 26: (D)
forms brilliant coloured dye wit h β -naphthol, the
compound under consideration must be p-toluidine
OH (c) as it is a primary aromatic amine. Primary aromatic
NO2 has –M and –I effect. p-Nitro-phenol more acidic amine, on treatment with NaNO2 in dil. HCl forms the
than ortho nitro phenol as o-nitro phenol intermolecular corresponding diazonium chloride salt.
H-bonding where p-nitro phenol has inter molecular
H-bond.
+
NH2 N2Cl-
NaNO2
H3C HCl H3C Chem i str y | 25.63

NH2
+
N2Cl- Sol 9:
O
NaNO2 Base
HCl NH2+ COCl NH C
H3C H3C

OH
Sol 10:

N N CH3 CH3
 -naphthol N + HNO2 No reaction
CH3
OH
Tertiary amine does not react with nitrous acid.

Sol 4: A → p; B → q; C → r; D → s
Sol 11: (B)
Sodium fusion extract gives Prussian blue colouration,
nitrogen and carbon both present in the compound. O
Phenolic group and salt of carboxylic acid gives FeCl3 conc.HNO3

:
N C
test. Chloride salt gives white precipitate of AgCl on conc.H2SO4
treatment with AgNO3. H
Hydrazone formation occur effectively at pH = 4.5 O
The reaction proceeds in that condition only when H+

:
concentration is just sufficient to activate the following O2N NH C
enolization.
O OH Due to presence of lone pair of electron on+ nitrogen
O
+
C
H
C+ atom, it will activate the ring
(A) and it -
will stabilize
N
intermediate cation at o and p positions.
O
As H+ concentration rises sufficiently, a large number -
Hence (B) is correct
of molecules of hydrazine gets converted into - O
hydrazonium ion which is not nucleophilic and reaction +
N
becomes impossible. Further low concentration of H+ Sol 12: (A) Same charges (B)
are present
+ at nearest
(in the case of 2,4-dinitrophenylhydrazinium bromide) position (Less stable) O
-
is not effective to proceed elimination. O
+ O -
+ +
N (C) + N
Sol 5: Aniline: It is a stronger base than either phenol (A) -

or nitrobenzene. O O
-
-
O -
- O
Sol 6: Zwitter ionic: +
N +
(B) + (D) + N
+ O
- O
H3N SO-3 -
- O
sulphanilic acid +
⊕ ⊝
Sol
(C) 13:
+ (D) C6H5N2 Cl
N gives scarlet red coloured dye

CuCl
+ with β-naphthol O
Sol 7: C6H5NH2 + NaNO2 
HCl
→ C6H5 N2 Cl 

heat
→ -
0ºC
C6H5–Cl -
Sol 14: A → r; s B →
O p, q; C → p, q, r; D → p, s
+
(D) + N
Sol 8: C6H6 + conc.HNO3/conc.H2SO4 → C6H5–NO2
O
Zn−HCl -
 → C6H5–NH2 (aniline)
2 5 . 6 4 | Amines and Aromatic Compounds Containing Nitrogen

Sol 15: (C)


NO2 NH2

red hot Fuming HNO3/H2SO4 H2S/NH3 NaNO2


3CH  CH
iron 873K Selective reduction H2SO4

NO2 NO2
N2 HSO 4-
+
OH

Hydrolysis

NO2 NO2

N2 HSO -4
+
OH

Hydrolysis

NO2 NO2

OH OH OH OH
O2N O2N
+
conc.H2SO4 NO2 dil.H2SO4
60oC 
OH OH OH OH
SO3H SO3H

NO2 NO2 NHCOCH3 NHCOCH3 NHCOCH3

NO2
Sn/HCl CH3COCl conc.H2SO4 HNO3

SO3H SO3H
dil. H2SO4 

NH2 NHCOCH3
NO2 NO2
OH-

NO2 NO2 NO2 NO2 NO2

KMnO4 H3O+ SOCl2 NH3


-OH/

CH3 COO- COOH COCl CONH2


Chem i str y | 25.65

Sol 16: (C)

:
OH
:
OH
H3C H3C C=O
C=O
NaNO2/HCl
CH CH2 CH CH CH2 HC
+
H3C H3C
NH2 NN

H
+
H3C H3C O
CH CH2 CH COOH CH CH2 HC
H2O C=O
H3C H3C
OH

Sol 17: (A)


O O
H3C H3C C H H3C C H
(i) O3 :
NH3
(ii) Zn.H2O +
CH2 C H CH2 CH NH3
O O-

OH O- O
H3C H3C + H3C C H
NH NH2
CH

:
OH CH2 OH CH2 CH NH2
-2H2O OH

OH
H3C
N

(S)

Sol 18: (A)


N=N-Ph
+ -
NH2 N2Cl OH
NaNO2, HCl -Napthol/NaOH
o
0C

(V) (W)
2 5 . 6 6 | Amines and Aromatic Compounds Containing Nitrogen

Sol 19: (B)


O
NH2 NH C CH3
O O
Pyridine
+ CH3 C O C CH3
-CH3COOH

Br2 + Acidic medium


O
Br N2Cl NH2 NH2 C CH3
+
Cu+HBr NaNO2+HCl H3O
273-278 K 

Br Br Br Br

Sol 20 and 21 (A) and (B):


Pr COOH COONH4
KMnO4 NH3
H+
Pr COOH COONH4
(P)

-2H2O

NH2 CONH2
Br2/NaOH

NH2 CONH2
(R) (Q)

Heat
Br
O O O
CH3 CH C O Et
C CH3 C C
O - + KOH
N CH N K N H
-H2O
C COOEt C C

O O O
+
H3O

COOH
+ CH3 CH COOH + Et OH
COOH NH2
(T)
Alanine

You might also like